You are on page 1of 67

C

Journey to Professionalism

TOPICS
1. Professionalism and the Clinical Medical Assistant
2. Developing Your Professional Persona

ABHES Content
VI.A.1.b.7.e.2

Professional conduct and appearance

CAAHEP Content
III.C.1.h.1
III.C.1.h.2
III.C.1.h.3
III.C.1.h.4

Professional attributes
Job readiness
Workplace dynamics
Allied health professions and credentialing

ONLINE COMPANION MATERIAL FOR THIS CHAPTER


2008 CAAHEP Content and Competencies Correlation Chart
Detailed chapter outline
Electronic competencies

Chapter updates
Access the Online Companion at www.delmarlearning.com/companions. Select Allied Health from the left menu, and then click on the link for
this book.

ACTIVITIES FOR THIS CHAPTER


Invite professionals from the industry to speak to your students about the importance of possessing professional qualities.
(Using members of your advisory committee, medical advisor, and externship coordinators to emphasize these important
qualities is a wonderful way to get students to buy in to what you are selling.)
397

27195_05_Lp01-09_395_452.indd 397

11/10/08 5:15:44 PM

398

Part 2 Chapter Lesson Plans

Invite former students to talk to the class about the importance of professionalism and to share the changes that they made
both personally and professionally to help them gain success in the industry.
Assign each student a professional partner so they can grade each other in professional categories such as: attendance,
appearance, attitude, motivation, empathy, etc. This assignment can be performed daily, weekly, or monthly and students
should change partners on a regular basis. (Emphasize the importance of using tact in areas where partners need to
improve and the importance of using praise in areas where partners are strong.)
Have students research ads in the newspapers and on the Web that list specific character traits or professional qualities that
are essential to possess in order to qualify for the position. Post these ads on a bulletin board and highlight these essential
qualities. This will serve as a reminder of the importance of professionalism.
Simulate performing a telephone reference on a past student who had poor attendance or other character flaws. Have
another instructor or student play the role of the employer asking questions such as: Tell me about the graduates attendance and dependability? Would you say that this individual is positive? Would you hire this graduate if you were an office
manager?
It will be very important for you to show how uncomfortable it is for the instructor to select the appropriate wording that will assist the graduate in obtaining the position while also being completely truthful. (Help students understand
that employers will not make future calls to the school if they feel that staff members providing references were dishonest
during a previous reference call.) Next simulate performing a telephone reference on a student with excellent professional
skills. This really helps the students see that they are writing their own references throughout the program and will hopefully inspire them to perform well throughout their training.
Using Table 1-2, have students highlight professional organizations with which your school is associated. Assign students
a project for exploring the Web sites of professional organizations that are endorsed by your school. Give each student a
different assignment for gathering specific information from each Web site, such as downloading a certification/registration application, a membership form, or a listing of test centers. Have students share their findings in a group or with the
whole class.
Have students prepare a credentialing plan that includes credentials that they would like to obtain upon graduation from
the program. Have them list what they will need to complete in order to be eligible for credentialing and a timeline for the
necessary items.
Have students attend some local or state chapter meetings and write a paper that outlines their experience.
Create a test or quiz using the test bank questions from Chapter 1 on the Instructor Resources CD.

STUDENT PREPARATION AND ASSIGNMENTS


Read Chapter 1 in the text and complete the chapter assessment questions, field scenario questions, and StudyWARE
Challenge.
Watch the DVD Link and complete the questions at the end of the chapter.
Assign any StudyWARE activities and games not included in the StudyWARE Challenge.
Complete the Chapter 1 Workbook activities.

DEVELOPMENTAL OBJECTIVES
After completing this chapter, you should be able to:
1. Correctly spell and define the essential terms.
2. Explain why a medical assistant working in a clinical capacity may be held to a higher degree of professionalism than a
medical assistant who works in an administrative capacity.
3. List six character qualities that are essential in a professional medical assistant.
4. List five external actions that must be taken in order to expand technical knowledge, communicate more effectively,
and demonstrate a caring attitude toward the patient.

27195_05_Lp01-09_395_452.indd 398

11/10/08 5:15:45 PM

Chapter 1 Journey to Professionalism

399

5. List two different types of communications and describe ways that overall communication skills can be improved.
6. Explain the importance of service when working as a medical assistant.
7. List four organizations that credential medical assistants.
8. List three other credentials that medical assistants may be able to obtain with some additional coursework or field
experience.

TEACHING AIDS AND EQUIPMENT


Computer to play DVD clips and games from the StudyWARE
Telephone (simulated reference call)
Multimedia projector and the Chapter 1 instructor slides created in PowerPoint found on the Instructor Resources CD
Test bank for Chapter 1 found on the Instructor Resources CD

REFERENCES AND RESOURCES


Thomson Delmar Learnings Critical Thinking for Medical Assistants DVD Series. Program 1, Professionalism and Career
Planning (Series ISBN 1-4018-3857-X)
How to Get a Job in Health Care, Zedlitz (ISBN 0-7668-4193-6)

WEB SITES
American Association of Medical Assistants: www.aama-ntl.org
American Medical Technologists: www.amt1.com
National Healthcareer Association: www.nhanow.com
National Center for Competency Testing: www.ncctinc.com

ANSWERS TO CRITICAL THINKING CHALLENGE TOOL BOX


Note: Answers may vary from those suggested below.
You are about a third of the way into your training program. Your instructor has repeatedly stated that a reference is not a gift,
but rather something that you have to earn. Your attendance has not been very good up to this point. You are now really starting to see the value of a good reference.
1. Is there anything that you can do to salvage your reputation with your instructor at this point in the program?
A suggested response may be: I know that I cant erase my absences; however, I can change
what occurs from this point throughout the remainder of the program by striving to be here
every day. I am hopeful that my actions will validate to the instructor that I am honestly trying to
change for the better and that the instructor will start to rebuild confidence in me.
2. Do you think that your instructor should bend the truth with potential employers regarding your early attendance problems as long as you correct your attendance for the remainder of the program? Why or why not?
A suggested response may be: No, I know that the instructor can not bend the truth about my
early attendance problems; however, I am hopeful that the instructor will be able to tell potential
employers about the wonderful progress I made throughout the remainder of the program and
that it appears at this point that I would be a wonderful employee.

27195_05_Lp01-09_395_452.indd 399

11/10/08 5:15:45 PM

400

Part 2 Chapter Lesson Plans

ANSWERS TO THE FIELD APPLICATION CHALLENGE


Note: Answers may vary from those suggested below.
You are a medical assistant who works for a midsize family practice. While you are cleaning the examination room, your
coworker, Sarah, enters the doorway and asks to speak with you for a moment. Sarah tells you that she believes that another
coworker, Kim, lied about being ill today and that the only reason Kim didnt come in is because she is lazy and doesnt want
to help on file purging day. Sarah ends the conversation by telling you that she doesnt mean to gossip but that she really
feels that Kim is a poor worker and that she is tired of picking up the slack for her. You personally have always found Kim to
be quite helpful and honest.
1. Do you think that it is right for Sarah to voice her concerns about Kim to another coworker?
It is best to avoid talking to employees about other employees. Sarahs comments are purely speculations. This type of behavior spreads negativity throughout the office.
2. What is an appropriate way to respond to Sarahs accusations?
Tell Sarah that you do not feel comfortable talking about another staff member and excuse yourself from the area.
3. Do you feel that you should share this information with Kim when she returns? Explain your response.
It is best not to say anything to Kim. This will cause tension in the office and promote negativity.

ANSWERS TO THE CHAPTER ASSESSMENT


1. The ability to place yourself in someone elses shoes or situation is known as:
A. sympathy.
B. empathy.
C. imitation.
D. projection.
2. All of the following are keys to unlocking the door to professionalism except:
A. attitude.
B. education.
C. appearance.
D. talent. (Talents are gifts that you either have or dont have. Even though you can develop certain talents, you cant
develop a talent that you dont have. However, professional qualities are attainable characteristics that anyone can
develop by making a conscious decision to improve in professional areas.)
3. Which of the following does not contribute to a professional work environment?
A. Gossip
B. Initiative
C. Helpfulness
D. Integrity
4. While serving during your externship, you observe Sandyone of the employees at the externship siteslip an ampule
of Demerol into her lab jacket. She doesnt record it into the narcotics log and has no idea that you saw her take it. The
next day the office supervisor calls a meeting and tells everyone that the narcotics count was off last night and that they
are missing some ampules of Demerol. What of the following is the best course of action?
A. Tell Sandy that you saw her take the Demerol. (Telling Sandy that you saw her take the Demerol gives you a chance
to start a dialog with her and encourages her to go forward. The punishment may not be as severe if Sandy goes forward herself; however, if Sandy does not go forward then you should go to the supervisor.)
B. Tell one of the other staff members that you witnessed Sandy take the Demerol.
C. Tell the supervisor that you witnessed Sandy take the Demerol.
D. Say nothing at all.

27195_05_Lp01-09_395_452.indd 400

11/10/08 5:15:45 PM

Chapter 1 Journey to Professionalism

401

5. Each of the following organizations credential medical assistants except the:


A. AAMA.
B. ASCP.
C. NCCT.
D. AMT.
6. Which one of the following organizations does not provide a credential in phlebotomy?
A. AMT
B. NCCT
C. ASPT
D. AAMA
7. What is the only credentialing organization for medical assistants that does not require testing for credentialing?
A. ARMA
B. AAMA
C. AMT
D. NCCT

ANSWERS TO THE DVD LINK


Note: Answers may vary from those suggested below.
1. What character traits does Dee lack that will prevent her from being a successful professional if she doesnt make changes
before graduating?
Dependability and a good attitude
2. Are there any similarities between you and Dee?
Answers will vary.
3. Do you think that Dees instructor is going to be able to give her a good reference based on what you saw in the clip?
Answers will vary. With it being so late in the process, it is going to be difficult for the instructor
to give Dee a good reference. Hopefully the interaction with the instructor will help Dee make
changes that will assist her in performing well during her externship. A successful externship will
give Dee at least one good reference.
4. What changes do you need to make in order to be a successful professional?
Answers will vary.

27195_05_Lp01-09_395_452.indd 401

11/10/08 5:15:45 PM

27195_05_Lp01-09_395_452.indd 402

11/10/08 5:15:45 PM

Organization and Time


Management in the
Medical Office
TOPICS
1.
2.
3.
4.
5.
6.

Becoming Organized While You Are in School


Getting Acclimated to Your New Work Environment
Setting Up Your Workstation and the Clinical Area
Daily Procedures That Clinical Staff Members Perform
Clinical Assisting and Time Management Issues
Performing Routine Maintenance on Clinical Equipment

ABHES Content
VI.A.1.b.9.c.6
VI.A.1.b.9.d

Medical office clinical procedures: housekeeping


Medical office clinical procedures: housekeeping; medical
equipment

ABHES Competencies
VI.A.1.a.6.b
Operate and maintain facilities and perform routine maintenance
of administrative and clinical equipment safely
CAAHEP Competencies
III.C.3.c.4.b
Performing routine maintenance on clinical equipment

ONLINE COMPANION MATERIAL FOR THIS CHAPTER


2008 CAAHEP Content and Competencies Correlation Chart
Detailed chapter outline
Electronic competencies
Chapter updates
Access the Online Companion at www.delmarlearning.com/companions. Select Allied Health from the left menu, and then click on the link for
this book.
403

27195_05_Lp01-09_395_452.indd 403

11/10/08 5:15:45 PM

404

Part 2 Chapter Lesson Plans

NOTES TO THE INSTRUCTOR


Organization and time management are skills that are difficult to master for most individuals starting something new. They are
especially difficult areas for individuals who struggle with organization in general. This chapter will assist students with getting
the most out of their learning and provide them with tips on organizing their study environment as well as tips for staying
organized while working in the industry.

ACTIVITIES FOR THIS CHAPTER


Have students purchase a planner or large desk calendar to help them remember important dates. (A large desk calendar was suggested in the text because of how visible it is.) If students prefer using electronic calendars, take students
into the computer lab and demonstrate how to use an electronic calendar as well. Spend some time during one of the
first days of class in helping students set up a study/assignment matrix that helps them manage their study time.
Have a former or senior student provide new students with tips for staying organized and on task while going through
the program.
Form groups and have each group come up with its own tips for studying, time management, and getting the most out
of the educational experience.
Have an organization coach put on a workshop for your students.
Have students interview an office manager or medical assistant in the field who can provide tips for workplace organization and time management.
Have a medical supplier come in and provide tips for ordering supplies when in the industry and expound on cost cutting tips and rewards incentives.
Have students design an inventory form for lab supplies, perform monthly inventories, and help check in lab supplies
as they come in. This will help students once they enter the industry and will also help you save time as well.
Use the instructor slides created in PowerPoint found on the Instructor Resources CD to deliver the chapter material.
Create a test or quiz using the test bank questions from Chapter 2 on the Instructor Resources CD.

STUDENT PREPARATION AND ASSIGNMENTS


Read Chapter 2 in the text and complete the chapter assessment questions, field scenario questions, and StudyWARE
Challenge.
Watch the DVD Link and complete the questions at the end of the chapter.
Assign any StudyWARE activities and games not included in the StudyWARE Challenge.
Complete the Chapter 2 Workbook activities.
Complete Procedure 2-1: Performing Routine Maintenance on Clinical Equipment.

DEVELOPMENTAL OBJECTIVES
After completing this chapter, you should be able to:
1. Correctly spell and define the essential terms.
2. List different ways that the medical assistant can organize a study area at home to make it more efficient.
3. Describe how to properly prepare for the performance of clinical competencies.
4. List three ways that the medical assistant can become better acclimated to the medical office.

27195_05_Lp01-09_395_452.indd 404

11/10/08 5:15:45 PM

Chapter 2 Organization and Time Management in the Medical Office

405

5. Explain the importance of reading the office policy and procedures manual.
6. List supplies that can help to create and maintain an organized workstation.
7. Discuss the importance of setting up a pending workstation.
8. Discuss the importance of dividing the pending file workstation into specific categories.
9. List steps that can be taken before patients arrive to save time when patients are present.
10. List common procedures for opening and closing the office.
11. Describe the importance of creating and maintaining a daily task list.
12. List common tasks associated with working the floor of a medical office and to classify where each task falls on the
priority schedule.
13. List steps for maintaining equipment in the office.

TEACHING AIDS AND EQUIPMENT


Computer (to show how to set up an electronic calendar)
Multimedia projector and the Chapter 2 instructor slides created in PowerPoint found on the Instructor Resources CD
Notebook (for reference purposes with inventory forms, purchase orders, packing slips, and monthly billing statements)
Example of an office policy or procedures manual
A variety of supply and equipment catalogs
Test bank for Chapter 2 found on the Instructor Resources CD
Refer to the text for equipment/supply list for each procedure

REFERENCES AND RESOURCES


Getting Things Done: The Art of Stress Free Productivity by David Allen, Penguin Books (ISBN 0-14-200028-0)

WEB SITES
StudyTips.org: www.studytips.org (Great for student study tips)
StudyMinder: www.studyminder.com/flashcards.html (Great Web site; allows students to make electronic flashcards
and a whole array of other activities. There is a fee for using this Web site, but it is spectacular for students who really
struggle with learning.)
Organizational Tips Blog: http://organizational-tips.fremont-online.com (Great article on organizing the medical office)

ANSWERS TO CRITICAL THINKING CHALLENGE TOOL BOXES


Note: Answers may vary from those suggested below.
Critical Thinking Challenge # 1:
Your school does not provide a certification review book or does not distribute one until the end of the program. What are
some options for you to help you review for the test while you are still in school?
Treat every test as though you are studying what is necessary to learn for certification testing.
Many students question the teacher about what is going to be on the test. Dont learn for the
testlearn for life. Review the whole chapter, not just what you think will be on the test. If your
school does not issue a certification review book, you may consider purchasing one while you

27195_05_Lp01-09_395_452.indd 405

11/10/08 5:15:45 PM

406

Part 2 Chapter Lesson Plans

are going through the program. Many publishers have review books for sale. Review the chapter in the certification review book that coincides with the material you are currently learning in
class. Highlight information that you learn in class in the review book. That way, when you start
reviewing for certification or registration testing, you can concentrate first on what you didnt
cover in class and go back and review what you did cover after that. Even if your school does
issue a review book at the end of your program, you may consider purchasing a different review
book from the one the school distributes. This will give you two different review books to study
from and will allow you to highlight information within the first review book for future studying.
Critical Thinking Challenge # 2:
It is the end of the day and everyone has gone home except for you. The provider has finished going through the files in the
pending file workstation and has left the office. You still have a couple of callbacks to perform. You are trying to reach a patient
who had a Pro-Time (a lab test that measures how well the blood is clotting). The Pro-Time is at a critical level and the provider
has instructed you to call the patient and have her change the amount of Coumadin (blood thinning medication) that she is taking. You have tried calling the patients home phone number but no one is home. There is an answering machine, but the privacy statement does not specify that you can leave results on the answering machine. The patient does not have a cell phone.
1. What are your options?
You can leave a message on the answering machine requesting the patient to call the office as
soon as possible. Second, if the patient does not call back, you should try to contact the physician
on call or the provider who gave the order and ask for instructions on how to proceed. The provider may try to reach the patient him- or herself, or may tell you that the call can wait until the
next morning. (Never leave a critical lab sitting on a desk. The ramifications of not calling could
put the patient at risk for complications.)
Critical Thinking Challenge # 3:
You work for a provider who has three exam rooms. You are working with a patient in room 1 when the provider knocks on
the door and tells you that she needs you to assist her with a Pap exam on the patient in room 2. You still need to complete the
workup on the patient in room 1 and room 3 is empty.
List the order in which you will perform the tasks:
2
3
1

Finish working up the patient in room 1.


Put a patient in room 3.
Assist the provider in room 2.

It is obvious that you are now behind; how might you get caught up?
You may ask a coworker to cover your rooms while you are assisting the provider; this will prevent you from running behind in the first place or after finishing the procedure. However, this is
possible only if your coworker is caught up with his or her patients. If not, just work as efficiently
as possible to get caught up, remembering the order in which things must be done. Always keep
rooms filled so that the provider is not waiting for another patient.

ANSWERS TO THE FIELD APPLICATION CHALLENGE


Note: Answers may vary from those suggested below.
You are a relatively new medical assistant. You notice that both you and your provider appear to be the last ones to leave every
day. You rarely get to take a lunch or any other breaks and it seems that your provider is always waiting on you to finish up with
patients so that she can perform the examination. The provider has been very nice, but you sense that she is becoming more
agitated because of continually running behind schedule.
1. What may be some contributing factors for always running behind?
Answers will vary; example may include: It could be the newness factor; however, it may be other
things as well, such as organization problems, spending too much time with the patients, not
understanding all of the facets of the position.

27195_05_Lp01-09_395_452.indd 406

11/10/08 5:15:45 PM

Chapter 2 Organization and Time Management in the Medical Office

407

2. How might you approach this with the provider?


Be open and honest with the provider. Tell her that you are concerned about running behind all
of the time and ask the provider if she has any pointers that can assist you with improving in this
area. This opens a line of communication between you and the provider and tells the provider
that you truly want to improve.
3. Is there anyone else that might be able to assist you?
Talk to coworkers and see if they have any tips. Of course, talking to your immediate supervisors
may also be very helpful.

ANSWERS TO THE CHAPTER ASSESSMENT


1. The term provider usually refers to a:
A. medical assistant.
B. receptionist.
C. physician, nurse practitioner, or physicians assistant.
D. LPN or LVN.
2. Instructions such as taking respirations on all patients with breathing disorders or putting anyone with chest pain into
the trauma room would be an example of office:
A. protocol.
B. triage.
C. assessment.
D. pending task.
3. Lab results may be given to a patients spouse, only if:
A. the patient gives verbal permission.
B. the patient has indicated on the privacy statement that the spouse may receive results.
C. the patient is unavailable.
D. the spouse agrees to tell the patient.
4. A mammogram would be an example of a:
A. yearly health requirement.
B. preventative health maintenance procedure.
C. physical maintenance.
D. none of the above.
5. When a medical assistant works in a clinical capacity, it is quite often referred to as:
A. working the front.
B. working the floor.
C. treating patients.
D. rooming patients.
6. You just worked up a patient in room 3. Dr. Jones comes out of room 2 and tells you that he needs an ECG performed on Mrs. Nasby in room 2. Dr. Jones now goes into room 3. Room 1 is open for a new patient. What should be
your first priority?
A. Rooming a patient for room 1
B. Run the ECG in room 2
C. Wait outside the door of room 3, just in case the provider needs you
D. None of the above

27195_05_Lp01-09_395_452.indd 407

11/10/08 5:15:46 PM

27195_05_Lp01-09_395_452.indd 408

11/10/08 5:15:46 PM

The Complete Medical


Record and Electronic
Charting
TOPICS
1. The Medical Record
2. Medical Record Formats
3. Contents of the Medical Record
4. Creating and Maintaining the Medical Record
5. Laws That Affect the Medical Record
6. Ownership, Retention, and Disposal of Medical Records

ABHES Content
VI.A.1.b.4.a
VI.A.1.b.7.a
VI.A.1.b.7.a.1
VI.A.1.b.8.a
VI.A.1.b.8.b

Ethical decisions, medical jurisprudence, and confidentiality


Manual and computerized records management
Patient case histories (confidentiality)
Office machines, computerized systems/medical data processing
Medical correspondence and medical reports

ABHES Competencies
VI.A.1.a.3.b
Prepare and maintain medical records
VI.A.1.a.2.n
Application of electronic technology
CAAHEP Content
III.C.1.c.1
Legal guidelines/requirements for health care
III.C.1.c.2
Medical ethics and related issues
III.C.1.f.1
Basic medical assisting clerical functions

409

27195_05_Lp01-09_395_452.indd 409

11/10/08 5:15:46 PM

410

Part 2 Chapter Lesson Plans

CAAHEP Competencies
III.C.3.c.2.c
Establish and Maintain the Medical Record
III.C.3.c.4.c.
Utilize Computer Software to Maintain Office Systems

ONLINE COMPANION MATERIAL FOR THIS CHAPTER


2008 CAAHEP Content and Competencies Correlation Chart
Detailed chapter outline
Electronic competencies
Chapter updates
Access the Online Companion at www.delmarlearning.com/companions. Select Allied Health from the left menu, and then click on the link for
this book.

ACTIVITIES FOR THIS CHAPTER


Have students make up charts that can be used throughout the program. Divide the charts both into administrative and
clinical sections. (It is best if students use hypothetical personal information for their address, phone numbers, billing
information, etc.)
Have students create an electronic medical record for themselves using the Synapse software in the back of their workbook.
Invite a guest speaker who works in the medical records department of a local hospital or clinic to discuss the latest guidelines in medical records.
Invite a guest speaker from a medical record supplier to demonstrate various supplies that are used for medical records
including: color-coded stickers, different styles of folders, different forms that can be designed for the clinic, and various
storage units.
Invite several EMR vendors to come to your school to demonstrate their EMR software.
Download trial versions of different EMR software from the Internet and share the various features with the class.
Create a test or quiz using the test bank questions from Chapter 3 on the Instructor Resources CD.

STUDENT PREPARATION AND ASSIGNMENTS


Read Chapter 3 in the text and complete the chapter assessment questions, field scenario questions, and StudyWARE
Challenge.
Watch the DVD Link and complete the questions at the end of the chapter.
Assign any StudyWARE activities and games not included in the StudyWARE Challenge.
Complete the Chapter 3 Workbook activities.
Complete Procedure 3-1: Create and Maintain the EMR Using SynapseEHR 1.0 Software.

DEVELOPMENTAL OBJECTIVES
After completing this chapter, you should be able to:
1. Correctly spell and define the essential terms.
2. List three different types of medical records and describe each one.
3. List important reasons for keeping neat, structured medical records.

27195_05_Lp01-09_395_452.indd 410

11/10/08 5:15:46 PM

Chapter 3 The Complete Medical Record and Electronic Charting

411

4. List the two major types of formats that are used for documenting in the patients record.
5. Describe the POMR approach and list the pros and cons for using this system.
6. Describe each letter of the SOAP format and list appropriate information to include under each section.
7. List administrative and clinical components of the medical record.
8. List common functionalities of the EMR and the benefits of this system.
9. List who is responsible for certifying EMRs.
10. Define HIPAA and give examples of ways that the office can become HIPAA compliant.
11. Determine which sections of the medical record are owned by the health care provider and which sections belong to
the patient.
12. Describe how long medical records have to be retained and how to properly dispose of them.

TEACHING AIDS AND EQUIPMENT


DVD player
Multimedia projector and the Chapter 3 instructor slides created in PowerPoint found on the Instructor Resources CD
Supplies for making up a medical chart: patient folders with dividers, color-coding stickers, forms, etc.
Computer and Internet for demonstrating different EMR software
Test bank questions for Chapter 3 found on the Instructor Resources CD
Refer to the text for equipment/supply list for each procedure

REFERENCES AND RESOURCES


Thomson Delmar Learnings Skills and Procedures for Medical Assistants DVD Series. Program 1, Administrative and Clinical
Sections of the Medical Record (Series ISBN 1-4018-3868-5)
Using the Electronic Record in the Health Care Provider Office, Eichenwald-Maki and Petterson (ISBN 1-4180-4988-3)

WEB SITES
Centers for Medicare and Medicaid Services: www.cms.hhs.gov/EducationMaterials/Downloads/HIPAA101-1.pdf
Problem-Knowledge Couplers: www.pkc.com/papers/pomr.pdf
American Health Information Management Association: http://library.ahima.org/xpedio/groups/public/documents/
ahima/bok1_036240.pdf
ComChart: www.comchart.com (Includes wonderful screen shots that illustrate EMR functions)
Misys: www.misys.com (Includes several short demos)
MedSphere Systems Corporation: www.medsphere.com (Good information about VistA software)
MedSphere Systems Corporation: www.medsphere.com/media/flash/revolution.html (Includes a phenomenal animation on the evolution of VistA and its personal VistA software product)
WorldVista: www.worldvista.org (Includes information about VistA-Office EHR software)
Certification Commission for Health Information Technology: www.cchit.org (To learn more about private-sector certification for HIT products)

27195_05_Lp01-09_395_452.indd 411

11/11/08 3:13:10 PM

412

Part 2 Chapter Lesson Plans

ANSWERS TO CRITICAL THINKING CHALLENGE TOOL BOXES


Note: Answers may vary from those suggested below.
Critical Thinking Challenge # 1:
The provider wants you to show the patient his blood sugar results for the last year and track what medications appear to be
most responsive to lowering his blood glucose.
1. What type of lab display would be best for this scenario?
A graphing display does a good job illustrating the peaks and valleys and is easier to visualize
than a series of numbers.
Critical Thinking Challenge # 2:
Mr. Walker calls the office and states that he heard a message on his answering machine that his wife needed to call the office.
He states that his wife is out of town and asks you to give the information to him.
1. Are you allowed to release the information to the husband?
No, not unless there is a written release from the patient or her husband is listed on the privacy
statement stating that PHI can be released to him.
2. Where in the chart can you find out if you can release the information?
It may be listed on the privacy statement or there may be a signed release form filed in the
administrative or legal section of the chart.
3. If after checking the documentation in the chart you find that you can release the information to the husband, how much
information should you release?
Just what is minimally necessary to handle the current concern.

ANSWERS TO THE FIELD APPLICATION CHALLENGE


Note: Answers may vary from those suggested below.
As a medical assisting student, you perform your externship in an office that uses EMR. You were able to see the wonderful
advantages of EMR and are hopeful that you will get a position in an office that uses EMR. Unfortunately, the office that hires
you still uses paper records. The physician has stated several times that he is hesitant to use EMR because of the software
cost.
Write a proposal to the physician stating why the office should go from paper records to EMR. To help you prepare,
complete the following:
1. On a separate sheet of paper, list at least 10 functions of EMR.
Answers will vary but may include any of the following:
Creates customized progress notes and builds notes efficiently through standard templates
and copy and paste features
Enables the provider and staff members to e-mail or fax progress notes, prescriptions, and
orders directly from the point of care
Allows team members to schedule appointments from the point of care
Automatically files and displays lab results in a variety of different formats
Graphs lab values, pediatric growth patterns, and vital signs
Displays several parts of the chart at one time
Allows multiple users access to different parts of the chart at the same time
Provides electronic tasking features to help keep staff members organized and greatly
improves time management

27195_05_Lp01-09_395_452.indd 412

11/10/08 5:15:46 PM

Chapter 3 The Complete Medical Record and Electronic Charting

413

Provides full remote access of patient records for those authorized to view them
Interfaces with the clinics practice manager program, making billing more efficient
Provides reporting and benchmarking capabilities that allow users to compare patient outcomes or to track other statistical data
2. List at least five advantages for using EMR.
Answers will vary but may include the following:
You never lose or misplace charts (time and money saver)
Reduces the risk of errors associated with illegible handwriting and the use of medical abbreviations
Greatly reduces the amount of time spent in performing duties associated with callbacks, finding the chart, writing and attaching a message to the chart and walking it back to the provider, the provider writing a message and attaching to the chart and walking it back to staff
members, calling pharmacies with prescriptions, etc.
Charting is much quicker through standardized templates and copy and paste features
Electronic messaging saves a great deal of time in writing and walking messages between
office staff members (money saver)
Reduces postal expenses because records and reports are sent electronically
Frees up space that is currently being used to store medical records
Will no longer need to store inactive files in a storage unit or use the professional services of
a medical record storage company (money saver)
3. Using an Internet search engine, look up three different EMR software vendors and price their software.
The answers will vary depending on which vendors students use. Students may have a difficult
time in obtaining prices through the Internet itself. They may need to obtain the telephone number of the vendors customer service department and call the vendor, explaining what they need
for their project.

ANSWERS TO THE CHAPTER ASSESSMENT


1. Important uses of the medical record include all of the following except:
A. means of communication.
B. statistical data.
C. payment data.
D. information to pharmaceutical companies.
2. Which format has no systematic cross-referencing of information?
A. POMR
B. Organizational
C. SOAP
D. SOMR
3. In the POMR format, the category that includes patient history, physician findings, and baseline results is the:
A. problem list.
B. database.
C. plan.
D. progress notes.

27195_05_Lp01-09_395_452.indd 413

11/10/08 5:15:46 PM

414

Part 2 Chapter Lesson Plans

4. Which organization has been approved to certify products from HIT vendors?
A. CCHIT
B. CMS
C. OSHA
D. HHS
5. Internal security measures to protect PHI include:
A. using encrypted passwords.
B. limiting account accessibility.
C. changing pass codes on a regular basis.
D. all of the above.
6. Which of the following would be considered a subjective finding?
A. Vital signs
B. Physical exam findings
C. Diagnostic test results
D. Patients chief complaint
7. Nonlaboratory test results should be filed in which section of the medical record?
A. Demographic section
B. Diagnostic reports
C. Progress notes
D. Lab reports
8. HIPAA helps protect:
A. PHI.
B. IHI.
C. PMI.
D. PPE.
9. In order to give personal health information to a relative, the patient must:
A. state the name of the individual on the privacy statement.
B. have written consent from the patient to share information.
C. both A and B.
D. none of the above

ANSWERS TO THE DVD LINK


Note: Answers may vary from those suggested below.
1. How did Irv try to prevent the patient from overhearing another patients information?
He excused himself when he saw Mrs. Nashel was eavesdropping on the conversation that he was
having with the physician and redirected Mrs. Nashel back up to checkout.
2. List other ways that Irv and the physician could have preserved the patients anonymity.
The conversation was taking place in an area where other patients could hear what was being
stated. It would have been best if the physician and the medical assistant had the conversation in
a closed area.
3. Do you agree with the way that the medical assistant addresses the physician? How would you address the physician?
The medical assistant addresses the physician as Doc. It would have been better for the medical
assistant to address the physician in a more formal manner, such as Doctor _______.

27195_05_Lp01-09_395_452.indd 414

11/10/08 5:15:46 PM

Fundamentals of
Documentation
TOPICS
1. Guidelines for Documenting in the Patients Chart
2. Making Corrections or Addendums to Chart Notes
3. Documenting and Sending Faxes
4. Writing and Sending E-Mails

ABHES Content
VI.A.1.b.3.d
VI.A.1.b.7.a
VI.A.1.b.7.a.1
VI.A.1.b.8.a
VI.A.1.b.8.b

Medical abbreviations
Manual and computerized records management
Patient case histories (confidentiality)
Office machines, computerized systems/medical data processing
Medical correspondence and medical reports

CAAHEP Content
III.C.1.e.4
III.C.1.e.5

Applications of electronic technology


Fundamental writing skills

ONLINE COMPANION MATERIAL FOR THIS CHAPTER


2008 CAAHEP Content and Competencies Correlation Chart
Detailed chapter outline
Electronic competencies
Chapter updates
Form pack
Access the Online Companion at www.delmarlearning.com/companions. Select Allied Health from the left menu, and then click on the link for
this book.

415

27195_05_Lp01-09_395_452.indd 415

11/10/08 5:15:47 PM

416

Part 2 Chapter Lesson Plans

NOTES TO THE INSTRUCTOR


Documentation within a patients medical record is very important because of the legalities associated with it. Students should
practice documenting throughout the entire program. Not only should students practice documentation with progress notes,
they should also have lots of practice with form completion.

ACTIVITIES FOR THIS CHAPTER


Have students make individual medical records that can be used throughout the entire program.
Create the following log books that can be used in the school laboratories (medical lab and pharmaceutical lab). If unable
to make up actual student medical records, have students purchase a notebook with dividers. Start out by having the students insert different dividers for each student in the classroom.
Within each divider, create the following sections:

Progress notes

Patient history

Patient lab results

Patient medications

Add additional dividers at the back of the notebook for global logs, which may include the following:

Immunization logs

Narcotics logs

Non-narcotics logs

Testing logs

Control logs

Temperature logs

Additional forms (Students will place all forms used throughout the program in this section of the notebook, including: patient registration forms, lab requisition forms, lab reporting forms, telephone encounter forms, progress
notes, medical history forms, etc.)
Each time a student works on other students within the class, she will use her clinical notebook and turn to the
section which has the name of the student that she is working on and record the related information. This makes
it very easy to check a students documentation in one area for the entire program. (Go to the Online Companion for
this book to find a complete set of forms that can be used throughout the program. The forms can be altered to include your schools
information.)

Have students complete the Synapse exercises in the back of the workbook if they havent already done so.
Students may use Synapse to create their own EMR that can be used throughout the program. Some instructors create
EMR records for all students in each class using just one Synapse CD. You can add several patients to this program. This
allows students to document electronically as well while working in the lab. (The computer and CD may be placed in an
area close to the lab so that the students can document the information once they are completed in the lab.)
Create a test or quiz using the test bank questions from Chapter 4 on the Instructor Resources CD.

STUDENT PREPARATION AND ASSIGNMENTS


Read Chapter 4 in the text and complete the chapter assessment questions, field scenario questions, and StudyWARE
Challenge.
Watch the DVD Link and complete the questions at the end of the chapter.
Assign any StudyWARE activities and games not included in the StudyWARE Challenge.
Complete the Chapter 4 Workbook activities.

27195_05_Lp01-09_395_452.indd 416

11/10/08 5:15:47 PM

Chapter 4 Fundamentals of Documentation

417

DEVELOPMENTAL OBJECTIVES
After completing this chapter, you should be able to:
1. Correctly spell and define the essential terms.
2. Provide documentation practices that will aid in the providers defense should the record be subpoenaed in a court
of law.
3. List 11 Documentation Dos and four Documentation Donts when documenting in the medical record.
4. Recognize and memorize abbreviations that are routinely used in medical documentation. Recognize and avoid using
abbreviations that appear on the Joint Commissions Do Not Use abbreviations list.
5. List items that should be recorded when documenting laboratory procedures, in-office procedures, medication procedures, prescription orders, patient education sessions, patient telephone calls, referrals, precertifications, outside procedures, and hospitalizations.
6. List the steps that should be taken when making a correction or an addendum to both a paper and paperless record.
7. Describe why information on the fax cover sheet is so important.
8. Describe proper etiquette guidelines that should be adhered to when sending professional e-mails.

TEACHING AIDS AND EQUIPMENT


DVD player
Multimedia projector and Chapter 4 instructor slides created in PowerPoint found on the Instructor Resources CD
Forms that can be used for documentation (progress forms, lab requisition forms, logs, etc.)
Computer (for Synapse software)
Test bank questions for Chapter 4 found on the Instructor Resources CD
Refer to the text for equipment/supply list for each procedure

REFERENCES AND RESOURCES


Using the Electronic Record in the Health Care Provider Office by Shirley Eichenwald-Maki and Bonnie J. Petterson
(ISBN 1-4180-4988-3)

WEB SITES
The Joint Commission: www.jointcommission.org/PatientSafety/DoNotUseList (Includes a listing of Do Not
Use Abbreviations)
InfoPlease: www.infoplease.com/ipa/A0862708.html (Includes a listing of common misspelled words)

27195_05_Lp01-09_395_452.indd 417

11/10/08 5:15:47 PM

418

Part 2 Chapter Lesson Plans

ANSWERS TO CRITICAL THINKING CHALLENGE TOOL BOXES


Note: Answers may vary from the ones suggested below.
Critical Thinking Challenge # 1:
You notice that some fellow employees are still using abbreviations listed on the Do Not Use abbreviation list when documenting entries within patient records.
1. What should be your course of action, if any?
Unless you are in a position of authority, you should not address the issue. It is up to the provider
and supervisor to address the issue, not you.
Critical Thinking Challenge # 2:
Instructors, if you have a two-stick rule for blood draws, you may want to modify this challenge. The two-stick rule is introduced in the phlebotomy
chapter.
While performing phlebotomy, you miss the patients vein on the first two attempts. You finally obtain a sample on the third
attempt. You dont want the provider to know that you missed the patients vein two separate times.
1. Regardless, why is it important for you to document failed attempts?
If a patient calls complaining of problems later, it is important to have this information documented in the chart for reference purposes. It may also help to identify veins that should not be used
because of repeated failures.
2. Write a sample documentation of how you would document this type of encounter.
Example:
10/12/2011
0800

Phlebotomy (vacutainer method) for a CBC and metabolic panel per Dr. Miller.
Failed attempts in the left arm (antecubital area of baslic and cephalic veins.
Veins appeared to roll even with firm anchoring.) Successful draw from the
right arm (antecubital area of the right cephalic vein). One red and lavender
tube sent to ABC laboratories, Acq. 21396. Megan Speck, CMA (AAMA)________

ANSWERS TO THE FIELD APPLICATION CHALLENGE


Note: Answers may vary from those suggested below.
You have had a busy morning and the physician has pulled you away from your documentation several times to assist with
special procedures. When you finally return to documenting, you find that you have transposed information for two different
patients into a patients chart. The physician has already documented information on the progress note and your information
is written below the physicians entry.
1. Would you rewrite both the physicians entry and your entry onto a new progress note so that it is neat and free of errors?
Why or why not?
No, you cannot rewrite any chart entry because it may appear that you are trying to cover something up in the event that the chart would to go to court.
2. If you wouldnt rewrite the entire progress note, what would you do to correct the errors?
Write the abbreviation CORR beside the error. Draw a single line through the error, date the error,
sign the error (some offices may just have you initial the error), and write in the correction.
3. What are some things you can do to avoid this type of scenario in the future?
Only document in one chart at a time. Separate the charts so that they cannot be mixed up.

27195_05_Lp01-09_395_452.indd 418

11/10/08 5:15:47 PM

Chapter 4 Fundamentals of Documentation

419

4. Would you alert the physician of the errors? Why or why not?
Yes, the physician should know that the two entries were transposed and that you corrected each
entry. Also explain how the error occurred and what you plan to do so that the error does not
reoccur. (Telling the physician about the error will give the physician a heads up so it wont be
such a big shock when the physician sees the error. The physician may be less likely to respond in
a negative manner because you explained how the mistake occurred and also discussed how you
will prevent this type of error from occurring in the future.)

ANSWERS TO THE CHAPTER ASSESSMENT


1. Which of the following would not be considered a Documentation Do?
A. Do document accurately.
B. Do include the date and time for each entry.
C. Do document subjectively.
D. Do use only standard abbreviations that have been approved by the practice.
2. Which of the following items should be included when documenting an in-house procedure?
A. The name of the procedure
B. The location where the procedure was performed (when applicable)
C. The storage cabinet from where you obtained supplies to perform the procedure
D. Both A and B
3. Which of the following would be harmful to the provider if the record went to court?
A. Documentation that was illegible and incomplete
B. An error that had been scratched out and re-written
C. A chart containing abbreviations that were on the Do Not Use abbreviations list
D. All of the above
4. Which of the following should be included when documenting a laboratory entry in the patients chart?
A. The name of the lab test
B. Where the specimens were sent
C. Any problems experienced during or following the procedure
D. All of the above
5. What items should not appear in a professional e-mail?
A. The name of the person to whom you are sending the e-mail
B. Emoticons
C. Common e-mail jargon used in personal e-mails such as lol
D. Both B and C
6. Which of the following should not be done when correcting an error in the chart?
A. Draw a single line through the error.
B. Record the date that you make the correction.
C. Start a new progress note and rewrite the entry so that the entry doesnt look messy.
D. Place your initials next to the error.

27195_05_Lp01-09_395_452.indd 419

11/10/08 5:15:47 PM

27195_05_Lp01-09_395_452.indd 420

11/10/08 5:15:47 PM

Conducting a Patient
Interview and Developing
a Medical History
TOPICS
1. Therapeutic Communication
2. Stages of the Patient Interview
3. Incorporating Effective Interviewing Techniques
4. Tools Used to Collect Medical History Information
5. Types of Health Histories
6. The Comprehensive Medical History

ABHES Content
VI.A.1.b.7.a
VI.A.1.b.7.a.1
VI.A.1.b.7.e
VI.A.1.b.8.a

Manual and computerized records management


Patient case histories (confidentiality)
Reception, public, and interpersonal relations
Office machines, computerized systems/medical data processing

ABHES Competencies
VI.A.1.a.4.a
Interview and record patient history
VI.A.1.a.2.f
Interview effectively
CAAHEP Content
III.C.1.e.1
Principles of verbal and nonverbal communication
III.C.1.e.2
Recognition and response to verbal and nonverbal communication
III.C.1.e.4
Applications of electronic technology

421

27195_05_Lp01-09_395_452.indd 421

11/10/08 5:15:47 PM

422

Part 2 Chapter Lesson Plans

CAAHEP Competencies
III.C.3.b.4.c

Obtain and record a patient history

ONLINE COMPANION MATERIAL FOR THIS CHAPTER


2008 CAAHEP Content and Competencies Correlation Chart
Detailed chapter outline
Electronic competencies
Chapter updates
Access the Online Companion at www.delmarlearning.com/companions. Select Allied Health from the left menu, and then click on the link for
this book.

ACTIVITIES FOR THIS CHAPTER


Have students perform at least three paper histories on different members of the class (information should be fictional
to protect each students health information).
Have students perform an electronic history using Synapse software after creating a chart for themselves.
Have students perform three family histories using the genogram method of documenting family histories.
Assign students to assist one of their family members in creating his or her personalized medical history form by going
to the DHHSs Web site, www.hhs.gov/familyhistory, and completing the electronic family history form. Due to confidentiality issues, the student should have the family member sign a sheet of paper stating that he or she created a personalized history by using the Web site, rather than making a copy of the information to share with the instructor. This
is a great form of patient education!
Create a test or quiz using the test bank questions from Chapter 5 on the Instructor Resources CD.

STUDENT PREPARATION AND ASSIGNMENTS


Read Chapter 5 in the text and complete the chapter assessment questions, field scenario questions, and StudyWARE
Challenge.
Watch the DVD Link and complete the questions at the end of the chapter.
Assign any StudyWARE activities and games not included in the StudyWARE Challenge.
Complete the Chapter 5 Workbook activities.
Complete Procedure 5-1: Conducting a Patient Interview and Completing a Patient History Form.

DEVELOPMENTAL OBJECTIVES
After completing this chapter, you should be able to:
1. Correctly spell and define the essential terms.
2. Explain how therapeutic communication improves the interviewing process.
3. Discuss how proxemics is influenced by culture and specific settings.
4. List three types of body language and discuss what role body language plays in communication.
5. List and describe the four stages of the patient interview.
6. Identify and describe three types of questioning techniques and list examples for each technique.

27195_05_Lp01-09_395_452.indd 422

11/10/08 5:15:47 PM

Chapter 5 Conducting a Patient Interview and Developing a Medical History

423

7. List and describe eight different types of ineffective questioning techniques and explain why they are ineffective.
8. List various senses that are used during listening and why active listening is so important during the interview process.
9. Identify and describe four different listening techniques and list examples of each technique.
10. List two methods that are used to collect patient data.
11. Describe four types of patient histories and describe when each method is used.
12. List and describe three parts of the medical history.
13. List five questions that should be posed when a patient answers Yes to a disease or condition during a health history.
14. Develop a family history using the genogram method.
15. Describe why the social part of the history may be more sensitive than other parts of the history.

TEACHING AIDS AND EQUIPMENT


DVD player
Computer and Synapse software to create a computerized history form
Computer with Internet connection to demonstrate the electronic medical history using the www.hhs.gov/familyhistory
Web site
Multimedia projector and the Chapter 5 instructor slides created in PowerPoint found on the Instructor Resources CD
Test bank questions for Chapter 5 found on the Instructor Resources CD
Refer to the text for equipment/supply list for each procedure

REFERENCES AND RESOURCES


Thomson Delmar Learnings Skill and Procedures for Medical Assistants DVD Series. Program 6, Taking a Patient History
(Series ISBN 1-4018-3868-5)

WEB SITE
U.S. Department of Health and Human Services: www.hhs.gov/familyhistory (electronic health history)

ANSWERS TO CRITICAL THINKING CHALLENGE TOOL BOXES


Note: Answers may vary from those suggested below.
Critical Thinking Challenge # 1:
The provider just revealed to the patient that his test results came back positive for cancer. The provider leaves the room and
the patient begins to cry. You lean over and give the patient a tissue. You take your hand and gently pat the patients shoulder.
The patient shrugs his shoulder and moves away from you.
1. What signal is the patient trying to send?
The patient wants to be left alone.
2. What would be an appropriate response from you at this point?
Nothing really needs to be said; a gentle nod of the head lets the patient know that you picked
up his signal.

27195_05_Lp01-09_395_452.indd 423

11/10/08 5:15:47 PM

424

Part 2 Chapter Lesson Plans

3. Is there anything that you can do to assist the patient?


At this point, you may ask the patient if he wants you to call anyone. If not, you should offer the
patient a chance to ask questions. You may review plans for follow-up with the patient at this
time. Giving the patient a packet of educational materials and the names of some support groups
may be helpful as well. Let the patient know that he is more than welcome to call the office with
any questions or concerns.
Critical Thinking Challenge # 2:
You are interviewing a 13-year-old male patient during the health history. His mother is in the room during the interview. You
notice that the young man is barely answering questions. Instead, his mother is answering. The patient appears frustrated that
his mother continues to stay in the room. The mother realizes she needs to run out to the car.
1. How could the mother staying in the room during the entire history impact therapeutic communication?
It could greatly interfere with therapeutic communication because a teenage son may not feel
comfortable in sharing information with his mother in the room.
2. What questions might you ask the patient while the mother is away?
Ask the patient if he feels uncomfortable with his mother in the room and ask him if there is anything that he wants to share while she is away.
3. How might you gently steer the mother to the reception area when she returns?
Tell the mother that you have a few things to finish up with the patient and you will call her back
to the room once the provider finishes the exam.

ANSWERS TO THE FIELD APPLICATION CHALLENGE


Note: Answers may vary from those suggested below.
You interview a Middle Eastern couple. The wife, who is the patient scheduled to have an immigration physical, can speak
English, but you notice that the husband is answering the majority of questions. The patient is quiet during the interview. You
do not feel as though you are connecting with her. At the end of the interview, you ask her to remove all articles of clothing
from the waist up. You notice that the husband becomes agitated. He asks you if the patient is seeing a female or male physician. You tell him that Dr. Jones is a male physician. The husband tells you that his wife will not be removing any clothes today
and that if you insist, they will leave.
1. Should you have asked the husband to leave during the patient interview? Why or why not?
No, it is quite common in some cultures for husbands to speak for their wives. Asking the husband to leave may cause frustration for all parties involved. You may, however, alert the physician of your observations. It will usually be up to the physician if he wants anything done
differently.
2. Could you have done anything differently to elicit more responses from the patient?
You may have tactfully made a statement such as the following: Thank you very much for assisting us during this phase of the questioning, however, for the remainder of the interview it may
be more helpful to have your wife answer the questions since the questions are related to her
personal health.
3. How should you respond when the husband refuses to allow his wife to disrobe?
I understand your hesitancy, sir. Let me talk to the physician to see if anything differently can be
done. (It is usually best if the physician handles this situation personally.)
4. Why do you think that the husband is reacting in the manner that he is?
Cultural concerns

27195_05_Lp01-09_395_452.indd 424

11/10/08 5:15:48 PM

Chapter 5 Conducting a Patient Interview and Developing a Medical History

425

ANSWERS TO THE CHAPTER ASSESSMENT


1. During the patient interview, the medical assistant notices that the patient moves his chair several inches further away
from where the medical assistant placed it. What should the medical assistant do?
A. Ask the patient to put the chair back where it was before.
B. The medical assistant should move his chair closer to the patient.
C. Do not acknowledge that the chair was ever moved. (The patient obviously does not feel comfortable where the
chair was placed. There is no need to make the patient feel awkward by asking questions about why he moved the
chair.)
2. What should you do if you notice that a Native American male patient will not look at you when responding to your
questions?
A. You should tell the patient that he needs to look at you when responding to your questions.
B. You should look away from the patient.
C. Since you recognize that this may be a cultural issue, you continue the questioning and minimize direct eye contact.
3. Which distance is considered acceptable for performing patient interviews?
A. Intimate distance
B. Personal distance
C. Social distance
D. Public distance
4. What is the average distance between the patient and the body of the medical assistant when performing blood
pressures?
A. Direct contact to 1.5 feet
B. 1.5 to 4 feet
C. 4 to 12 feet
D. Greater than 12 feet
5. The bulk of the data is collected during which stage of the interview?
A. The preparation stage
B. The greeting and introduction stage
C. The body of the interview
D. The conclusion
6. The type of questioning technique that is usually used at the beginning of the interview is referred to as:
A. open-ended questions.
B. close-ended questions.
C. silence.
7. Which of the following would be considered ineffective interviewing techniques?
A. Asking leading questions
B. Demanding an explanation
C. Strongly agreeing or disagreeing with the patient
D. All of the above
8. What is the name of the listening technique that stimulates the patient to ponder on their last response?
A. Clarification
B. Restating
C. Reflecting
D. Summarizing

27195_05_Lp01-09_395_452.indd 425

11/10/08 5:15:48 PM

426

Part 2 Chapter Lesson Plans

9. What type of family history is also considered a family tree?


A. Famogram
B. Genogram
C. Genofam
D. None of the above
10. What information would be included in the patients personal medical history?
A. Past medical history
B. Review of systems
C. Current medication listing
D. All of the above

ANSWERS TO THE DVD LINK


Note: Answers may vary from those suggested below.
1. What types of tasks should be performed to ready the exam room for the next patient?
Clean the exam room; replenish supplies; check to see why the patient is coming in and set up any
necessary trays, instruments, etc.
2. What should the medical assistant do to the Pap tray so that the patient is not distracted by the tray during the patient
interview or history?
Cover the tray or remove it from the patients view.
3. What additional form may need to be completed prior to the history form?
The patient may need to read and sign the practices privacy statement if one has not been signed
thus far.
4. What did you think of Anitas personality?
Anita was a bit dry (very serious); however, she was very professional as well. A light smile from
time to time could have made the patient feel more at ease.
5. What did Anita hand to the patient so that she could figure out her LMP?
A calendar

27195_05_Lp01-09_395_452.indd 426

11/10/08 5:15:48 PM

Developing In-Office
Screening Skills
TOPICS
1. Establishing Boundaries
2. The Role of the Medical Assistant during In-Office Screenings
3. Improving Anticipation Skills and Following Office Protocol
4. The Providers Role during the Assessment Process
5. The Follow-Up Appointment/Progress Note

ABHES Content
VI.A.1.b.9.c.2

Patient preparation

ABHES Competencies
VI.A.1.a.2.f
VI.A.1.a.4.ff

Interview effectively
Perform telephone and in-office screenings

CAAHEP Content
III.C.1.e.4

Applications of electronic technology

CAAHEP Competencies
III.C.3.b.4.a

Perform telephone and in-person screenings

427

27195_05_Lp01-09_395_452.indd 427

11/10/08 5:15:48 PM

428

Part 2 Chapter Lesson Plans

ONLINE COMPANION MATERIAL FOR THIS CHAPTER


2008 CAAHEP Content and Competencies Correlation Chart
Detailed chapter outline
Electronic competencies
Chapter updates
Access the Online Companion at www.delmarlearning.com/companions. Select Allied Health from the left menu, and then click on the link for
this book.

ACTIVITIES FOR THIS CHAPTER


Place cards listing chief complaints in a box and have each student who will be acting as the patient draw out a card
from the box. Have the same student act as though they have the symptoms on the card. The student who is role playing as the medical assistant should be able to develop the complaint correctly on the progress note, provide the patient
with proper disrobing instructions, and set the room up according to the patients symptoms.
Set up a clinical practical. Divide the classroom into stations or tables. Each station should have a variety of index
cards. The cards should include the following information: cards that describe a patients chief complaint, cards that
provide HPI information matching the chief complaint, cards with pictures of different positions and draping procedures, and cards that list or picture various supplies, instruments, or trays. (Include several extra cards for each category
that do not match the chief complaint. Give students a predetermined amount of time to match the symptom cards up
with the HPI information cards, correct positioning and draping cards, and supply and equipment cards.)
Using Synapse software, create several in-office screenings using the electronic medical records from different students.
Create a test or quiz using the test bank questions from Chapter 6 on the Electronic Classroom Manager CD.

STUDENT PREPARATION AND ASSIGNMENTS


Read Chapter 6 in the text and complete the chapter assessment questions, field scenario questions, and StudyWARE
Challenge.
Watch the DVD Link and complete the questions at the end of the chapter.
Assign any StudyWARE activities and games not included in the StudyWARE Challenge.
Complete the Chapter 6 Workbook activities.
Complete the procedures in this chapter.

Procedure 6-1: Conduct and Record Results from an In-Office Screening

Procedure 6-2: Conduct a Follow-Up Interview and Develop a Progress Note

DEVELOPMENTAL OBJECTIVES
After completing this chapter, you should be able to:
1. Correctly spell and define the essential terms.
2. Establish the boundaries of the medical assistant during in-office screenings.
3. List the parts of an in-office screening.
4. Explain how slightly developing the chief complaint can assist the medical assistant with anticipating how the patient
should disrobe and what instruments and supplies may be necessary during the exam.
5. Describe the providers role in the assessment process.

27195_05_Lp01-09_395_452.indd 428

11/10/08 5:15:48 PM

Chapter 6 Developing In-Office Screening Skills

429

6. State why the provider must develop her own HPI and list components of the HPI.
7. Describe the role of the medical assistant during the follow-up interview.
8. List and define the parts of a follow-up progress note.

TEACHING AIDS AND EQUIPMENT


DVD player
Computer and Synapse CD to create new office visit notes
Multimedia projector and the Chapter 6 instructor slides created in PowerPoint found on the Instructor Resources CD
Test bank questions for Chapter 6 found on the Instructor Resources CD
Refer to the text for equipment/supply list for each procedure

REFERENCES AND RESOURCES


Thomson Delmar Learnings Critical Thinking for Medical Assistants DVD Series. Program 6, Medical Assessment in the
Physicians Office (Series ISBN 1-4018-3857-X)
Delmars Clinical Handbook, Second Edition, Krebs and Heller (ISBN 1-4018-3285-7)

ANSWERS TO CRITICAL THINKING CHALLENGE TOOL BOXES


Note: Answers may vary from the ones suggested below.
Critical Thinking Challenge # 1:
While you are performing an in-office screening, the patient states that she is having horrific abdominal pain. The patient is
doubled over and clenching her abdomen. The pain came on suddenly, and the patient is also complaining of nausea, vomiting, and a fever. The patient tells you she is thirsty and asks you if you can get her some water. There are two other patients in
front of this one.
1. Should you talk to the provider about this patient before she goes in with the other two patients? If so, why?
Yes, severe abdominal pain, especially if acute, could be very serious.
2. Should you give the patient something to drink? Why or why not?
No, the patient should not receive anything to drink. Patients with acute abdominal pain may
need surgical intervention. Giving the patient something to drink could cause problems with the
anesthetic.
Critical Thinking Challenge # 2:
You are interviewing a 16-year-old female patient who is complaining of lower abdominal pain that radiates to her back side.
The pain has been present for three days but seems to be worsening. The pain level is about a 6 on the pain scale, according
to the patient. The patient also states that she has a green vaginal discharge that smells really bad.
1. How will you have the patient disrobe for the provider?
This patient should disrobe from waist down since the pain is lower and she also has vaginal
symptoms.
2. What tray might you set up just in case the provider wants to examine the patient more thoroughly?
A pelvic tray with cultures. The patient may have some type of a vaginal infection such as an STI
(sexually transmitted infection).

27195_05_Lp01-09_395_452.indd 429

11/10/08 5:15:48 PM

430

Part 2 Chapter Lesson Plans

Critical Thinking Challenge # 3:


The front office receptionist calls back to your extension to tell you that a patient walked in off the street complaining of chest
pain. The reception room is packed with patients that have scheduled appointments.
1. Should you take this patient straight back to the clinical area? If so, why?
Yes, the patient should be taken back right away. Chest pain can be a life-threatening condition
and time is very important.
2. What will you do if the patients from the reception area complain that this patient who didnt even have an appointment
was seen before them?
Tell the other patients that normally we see people in the order they arrive; however, this patient
had symptoms that warranted emergency intervention.
3. How should the patient be taken back to the exam or special procedures room?
By wheelchair.
4. What equipment should be available for this type of patient?
Pulse oximeter, crash cart, EKG unit, IV equipment and solution, and oxygen equipment
5. How quickly should the provider be notified? Should you start any procedures before the provider has had an opportunity
to examine the patient? Why or why not?
The provider should be notified ASAP. You may gather all of the equipment but should not start
any procedures without an order from the provider.

ANSWERS TO THE FIELD APPLICATION CHALLENGE


Note: Answers may vary from those suggested below.
A patient comes into your office with abdominal pain. You ask the patient to give the location of the pain, rate the pain, and
tell you how long the symptoms have been present. You dont ask any more questions and you have the patient disrobe from
waist up. The provider enters the room and develops the complaint. Upon expounding on the complaint, the provider finds
that the patient has urogenital symptoms in conjunction with the abdominal pain. Answer the following questions based on
the above scenario:
1. What mistakes did you make during the interviewing process?
You didnt develop the complaint enough to gain good anticipation skills.
2. Why should you perform a brief HPI, even through you may not be responsible for documenting the information?
If you do not properly develop the complaint, you will be unable to anticipate the needs of the
provider which will lengthen the patients visit causing everyone to run behind schedule.
3. What tray or supplies should you set up based on the providers findings?
Based on the patients symptoms, you should have set up a vaginal tray and collected a urine
sample (you do not test the urine; however, until you receive an order from the provider).
Performing these actions will save time for everyone. Remember that if the provider decides that
he or she does not need the pelvic tray or urine, you simply put away all of the items for the vaginal tray and dispose of the urine. It is better to be overly prepared than not have items that the
provider will need during the exam.
4. Did you have the patient disrobe appropriately? If not, how should you have had the patient disrobe?
No, the patient should have been asked to disrobe completely and given a gown or else asked to
disrobe from the waist down.

27195_05_Lp01-09_395_452.indd 430

11/10/08 5:15:48 PM

Chapter 6 Developing In-Office Screening Skills

431

ANSWERS TO THE CHAPTER ASSESSMENT


1. What question would not be necessary when screening a patient with chest pain?
A. Does the pain radiate to any other part of your body?
B. Are you experiencing any shortness of breath?
C. Do you have any dyspnea?
D. Are you experiencing any nausea or vomiting?
2. What is the job of the medical assistant during in-office screenings?
A. To gather the patients chief complaint
B. To perform vital signs
C. To diagnose the patient
D. Both A and B
3. Which of the following may not be performed by the medical assistant without an order from the provider?
A. Perform a blood glucose on the patient.
B. Set up a pelvic tray for a patient experiencing vaginal symptoms.
C. Have the patient disrobe from the waist down for anal symptoms.
D. Give a nauseated patient an emesis basin.
4. Which of the following does not need to be recorded when documenting a follow-up visit?
A. The date of the follow-up visit
B. The patients current medications
C. The patients compliance or noncompliance to follow home care instructions
D. The reason for the visit
5. Who is responsible for developing the HPI?
A. The provider
B. The medical assistant
C. Mainly the provider, but the medical assistant should ask enough questions to determine how to set up the patient
and the room
D. None of the above

ANSWERS TO THE DVD LINK


Note: Answers may vary from those suggested below.
1. In the first scenario, Keisha obtained the patients chief complaint and also developed the HPI. Do medical assistants
always develop the HPI when taking a patients chief complaint? Even if it is not the medical assistants role to record the
HPI, what HPI information would be helpful in determining how to prep the patient and set up the examination room?
Keisha did obtain a urine sample just in case the patient had a UTI. Should she run any testing on the urine before the
physician examines the patient?
Medical assistants do not always fully develop the HPI but should develop it enough to know
how to properly set up the patient and room for the exam. Knowing the related symptoms that
coincide with the complaint provides the information that is necessary to gain good critical thinking or anticipation skills. No, Keisha should not perform any testing on the urine until she receives
an order to do so from the physician.

27195_05_Lp01-09_395_452.indd 431

11/10/08 5:15:48 PM

432

Part 2 Chapter Lesson Plans

2. In the second scenario, Shanna asked few essential questions to help her anticipate how the patient should disrobe and
what trays may need to be set up. Additionally, she told the patient that she thought she probably had the flu. Why do you
think that the physician was so upset with Shanna? Do you feel that the physician was wrong to reprimand Shanna?
The physician was upset because Shanna gave the patient a possible diagnosis for her symptoms.
Additionally, Shanna didnt ask the appropriate questions to set up the room and patient properly, which now has prompted a delay in the schedule. The physician now has to wait for the
room and patient to be set up properly and for Shanna to collect the urine sample and test the
urine. Shanna needs to be reprimanded. Not only did she throw off the schedule for today, she
also gave a potential diagnosis for the patients symptoms which is practicing medicine without
a license. If Shanna continues these types of actions, they could cause legal problems for the
practice in the future.

27195_05_Lp01-09_395_452.indd 432

11/10/08 5:15:48 PM

Conducting Telephone
Screenings
TOPICS
1. Customer Service and Telecommunications
2. Triaging
3. Telephone Medicine
4. Screening Patient Test Results
5. Calling in Prescriptions
6. Sending Faxes
7. Electronic Mail (E-Mail)
8. Working with TDD or TTY Devices
9. Video Conferencing

ABHES Content
VI.A.1.b.7.e
VI.A.1.b.7.e.1
VI.A.1.b.8.a

Reception, public, and interpersonal relations


Telephone techniques
Office machines, computerized systems/medical data
processing

ABHES Competencies
VI.A.1.a.4.ff
Perform telephone and in-person screenings
VI.A.1.a.2.e
Use proper telephone techniques
VI.A.1.a.4.l
Screen and follow up patient test results

433

27195_05_Lp01-09_395_452.indd 433

11/10/08 5:15:49 PM

434

Part 2 Chapter Lesson Plans

CAAHEP Content
III.C.1.c.1
III.C.1.e.1
III.C.1.e.2
III.C.1.e.4
III.C.1.f.1

Legal guidelines/requirements for health care


Principles of verbal and nonverbal communication
Recognition and response to verbal and nonverbal
communication
Applications of electronic technology
Basic medical assisting clerical functions

CAAHEP Competencies
III.C.3.b.4.a
Perform telephone and in-person screenings
III.C.3.c.1.d
Demonstrate telephone techniques
III.C.3.b.4.i
Screen and follow up test results

ONLINE COMPANION MATERIAL FOR THIS CHAPTER


2008 CAAHEP Content and Competencies Correlation Chart
Detailed chapter outline
Electronic competencies
Chapter updates
Access the Online Companion at www.delmarlearning.com/companions. Select Allied Health from the left menu, and then click on the link for
this book.

ACTIVITIES FOR THIS CHAPTER


Invite some telephone vendors/office equipment vendors to display their phone systems/equipment and explain the
unique features of their systems to the class (telephones, fax machines, and TDDs).
Invite some medical assistants and supervisors from the field to talk to the class about the importance of phone etiquette and why good telephone communications is so important.
Set up stations with telephones and telephone screening/triage manuals. Have one student play the role of the medical
assistant while another student plays the role of the patient. Students should practice screening and documenting telephone calls.
Create a test or quiz using the test bank questions from Chapter 7 on the Instructor Resources CD.

STUDENT PREPARATION AND ASSIGNMENTS


Read Chapter 7 in the text and complete the chapter assessment questions, field scenario questions, and StudyWARE
Challenge.
Watch the DVD Link and complete the questions at the end of the chapter.
Assign any StudyWARE activities and games not included in the StudyWARE Challenge.
Complete the Chapter 7 Workbook activities.
Complete the Procedures in this chapter.

Procedure 7-1: Perform a Telephone Screening

Procedure 7-2: Screen and Follow Up on Test Results (Determine the Order of Prioritization)

27195_05_Lp01-09_395_452.indd 434

11/10/08 5:15:49 PM

Chapter 7 Conducting Telephone Screenings

435

DEVELOPMENTAL OBJECTIVES
After completing this chapter, you should be able to:
1. Correctly spell and define the essential terms.
2. Explain why customer service is so important when communicating with patients via telecommunications equipment.
3. Give four examples of different types of telecommunications devices.
4. Contrast the differences between telephone triage and telephone screenings.
5. List three different screening tools that can be used to screen patient phone calls.
6. List factors that must be considered when performing procedures via telecommunications.
7. List four components that should be included in a telephone screening training program.
8. List the desired traits of a telephone screener and three steps that should take place before the screening process begins.
9. List four different actions that are typically found in a telephone screening manual.
10. List several different types of life-threatening emergencies and explain two normal actions that are given to patients
with life-threatening emergencies.
11. Describe the medical assistants role in screening and calling patients with test results.
12. Explain how HIPAA laws impact both telephone screenings and telephone calls to patients about test results.
13. Explain the importance of critical or panic lab values.
14. Describe the following terms and explain how these types of telecommunications devices are used to communicate
with patients and other medical entities: TDD machines, fax machines, videoconferencing equipment, and e-mail.

TEACHING AIDS AND EQUIPMENT


DVD player
Computer and Synapse software to create telephone notes
Multimedia projector and the Chapter 7 instructor slides created in PowerPoint found on the Instructor Resources CD
Test bank questions for Chapter 7 found on the Instructor Resources CD
Various telecommunications equipment: fax machine, TDD, telephones, etc.
Examples of different telephone screening/triage manuals to pass around the class
Forms such as message pads, progress notes, telephone call forms, etc.
Refer to the text for equipment/supply list for each procedure

REFERENCES AND RESOURCES


Thomson Delmar Learnings Critical Thinking for Medical Assistants DVD Series. Program 6, Medical Assessment in the
Physicians Office (Series ISBN 1-4018-3857-X)
Delmars Clinical Handbook, Second Edition, Krebs and Heller (ISBN 1-4018-3285-7)

27195_05_Lp01-09_395_452.indd 435

11/10/08 5:15:49 PM

436

Part 2 Chapter Lesson Plans

ANSWERS TO CRITICAL THINKING CHALLENGE TOOL BOXES


Note: Answers may vary from those suggested below.
Critical Thinking Challenge # 1:
For each of the following responses, write T for triage responses and S for screening responses. Write TS if the response applies
to both.
S Let me transfer you to the physician.
T Based on your description, it sounds as though you may have shingles. I am going to set you up an appointment for
tomorrow.
TS To be on the safe side, lets have you evaluated by the emergency medical services.
TS I can schedule you for an appointment tomorrow at 10:00 a.m.
T You might try putting some ice on the ankle and giving us a call later today to report how you are doing.
Critical Thinking Challenge # 2:
A physician hands you a critical lab value result for a female patient with a low potassium level (an electrolyte important in heart
function). He asks you to contact the patient right away and gives you a list of instructions to give to the patient. It is Friday
evening and the office does not open again until Monday morning. The physician is late for a special dinner and leaves you all
alone. You call the patients home phone number only to find out that the number has been disconnected. The patient does
not list a cell phone number. You try to reach the patients daughter, who is listed as a contact, but there is no answer and no
voicemail. Its getting late and you want to go home.
1. What would be your most logical solution?
Try to reach the physician on call or the physician who gave the order and let him know that you
are unable to reach the patient. Ask the physician for instructions on how to proceed.
2. What are the possible consequences of waiting until the following Monday to contact the patient?
Because potassium is important in heart function, waiting even a few days could set the patient
up for a cardiac emergency.

ANSWERS TO THE FIELD APPLICATION CHALLENGE


Note: Answers may vary from those suggested below.
The husband of one of your patients calls to tell you that he just noticed that his wifes mouth is drooping on the left side and
that she is unable to use her left arm. He goes on to say that his wife appears to be a bit confused and has a terrible headache.
He wants to know if he should drive her to the office for an evaluation.
1. What do you suspect may going on with the patient?
A possible stroke
2. As a medical assistant, you do not have the authority to diagnose patients. But do you feel it is important that you recognize
symptoms of common emergency conditions so that you can instruct patients in the proper manner?
Medical assistants do not have the authority to diagnose patients but they should be able to recognize emergency symptoms so that they can direct the patient in the appropriate manner.
3. What would most office protocols typically suggest for someone experiencing this patients symptoms?
To hang up and dial 911 or the local EMS number ASAP.

27195_05_Lp01-09_395_452.indd 436

11/10/08 5:15:49 PM

Chapter 7 Conducting Telephone Screenings

437

ANSWERS TO THE CHAPTER ASSESSMENT


1. An example of a telecommunications device is a:
A. telephone.
B. fax machine.
C. computer modem.
D. all of the above.
2. Why is it considered wiser for the medical assistant to take on the role of a screener when talking to patients about
their concerns than to take on the role of a triager?
A. Taking on the role of a screener forces you to follow a protocol that was designed or approved by the provider and
to obtain clarification in cases that do not match the protocol book.
B. Screening patient calls gives you more freedom to use your own assessment skills and has a higher degree of responsibility than triaging.
C. Taking on the role of a triager requires more decision-making and could put you at risk for practicing medicine without a license.
D. Both A and C
3. This type of screening tool lists specific symptoms under a list of appointment actions:
A. screening grid.
B. algorithm.
C. appointment grid.
D. none of the above.
4. Before screening patient concerns over the telephone, the medical assistant should:
A. ask for a raise because of the liability ensued with screening patient calls.
B. go through a training program to learn how to properly screen patient calls.
C. use an approved protocol manual whenever screening patient calls.
D. both B and C.
5. What are desired traits of a good telephone screener?
A. A person who loves to talk on the phone
B. A person who hates working the floor
C. A person who has wonderful communication and documentation skills
D. A person who has a license to practice medicine
6. In regard to HIPAA, medical assistants can share patient information when:
A. Medical information directly pertains to the persons involvement with the patient or patients care.
B. You speak to the spouse of a patient.
C. The patient has listed the caller as someone that you can share information with on the privacy disclosure statement.
D. Both A and C.
7. When talking to a patient who is experiencing a life-threatening condition, you should always:
A. have the patient call the EMS.
B. notify the provider.
C. follow the office protocol for handling a life-threatening event.
D. follow your gut feeling.

ANSWERS TO THE DVD LINK


Note: Answers may vary from those suggested below.
1. In the first scenario, what was the reason that Jae Min didnt pull the patients chart herself when talking to Mrs. Hernandez
on the phone?
Because the patients symptoms were potentially life-threatening and every second counts.

27195_05_Lp01-09_395_452.indd 437

11/10/08 5:15:49 PM

438

Part 2 Chapter Lesson Plans

2. We saw Jae Min ask Mrs. Hernandez several questions so that she could determine if Mrs. Hernandez was in the middle
of a life-threatening emergency. Do you think that there was a need to ask the patient so many different questions when it
was clear that the patient was in the middle of an emergency early in the call?
It really depends on office protocol; however, the more quickly you can get help for the patient,
the better the patients chances are for a full recovery.
3. In the second scenario, the medical assistant scheduled the patient for a 3:00 p.m. appointment. Why was scheduling the
patient for an appointment an inappropriate response?
Because the patients symptoms pointed to a life-threatening event and any delay in quick evaluation and treatment could cost the patient her life.

27195_05_Lp01-09_395_452.indd 438

11/10/08 5:15:49 PM

Assisting Patients with


Special Needs
TOPICS
1. Legal Issues and Special Needs Patients
2. Working with Patients with Special Needs

ABHES Content
VI.A.1.b.5.b
VI.A.1.b.5.h
VI.A.1.b.7.e

Caring for patients with special and specific needs


Hereditary, cultural, and environmental influences on
behavior standards
Reception, public, and interpersonal relations

ABHES Competencies
VI.A.1.a.2.m
Adaptations for individualized needs
VI.A.1.a.7.b
Instruct patients with special needs
CAAHEP Content
III.C.1.c.1
III.C.1.d.3
III.C.1.e.1
III.C.1.e.2
III.C.1.e.3

Legal guidelines/requirements for health care


Hereditary, cultural, and environmental influences on
behavior
Principles of verbal and nonverbal communication
Recognition and response to verbal and nonverbal
communication
Adaptations for individualized needs

439

27195_05_Lp01-09_395_452.indd 439

11/10/08 5:15:49 PM

440

Part 2 Chapter Lesson Plans

CAAHEP Competencies
III.C.3.c.3.b

Instruct individuals according to their needs

ONLINE COMPANION MATERIAL FOR THIS CHAPTER


2008 CAAHEP Content and Competencies Correlation Chart
Detailed chapter outline
Electronic competencies
Chapter updates
Access the Online Companion at www.delmarlearning.com/companions. Select Allied Health from the left menu, and then click on the link for
this book.

ACTIVITIES FOR THIS CHAPTER


Invite patients from different cultures to come in and discuss their views on various health care issues with the class.
Invite translators (deaf interpreters, interpreters for patients with limited English) to discuss techniques and methods
that can be helpful when working with patients from these populations with the class.
Invite a mental health worker to discuss with the class the techniques or methods that can be used when working with
patients who are mentally impaired or challenged.
Invite seniors to discuss with the class their expectations of health care workers and to provide tips for making office
visits the best they can be.
Put together a pediatric day during which students are allowed to bring in their children to class. (Children should be
accompanied by another qualified adult, so that the students can continue to work.) Have students practice noninvasive
procedures on the children such as vision and hearing exams, vital signs, height and weight, and properly identifying
landmarks where injections should be administered. This could be an event in which stations are set up to perform
each set of procedures. Assign students to write a paper regarding how they felt as they worked with the children. They
should also include their thoughts on what they gained from this activity and what they may do differently when working with children in the future.
Have students divide into groups. One group of students will act as medical assistants and the other group of students
will act as though they are blind by putting on blindfolds so that they are unable to see anything. Each student playing the role of medical assistant should practice assisting one of the students playing a blind patient from the reception
room to the examination room as well as ascending and descending a staircase, going through a door, and sitting in a
chair.
Assign students to write a research paper regarding the Americans with Disabilities Act and describe how this act protects patients who are blind, hearing impaired, mentally disabled, or physically disabled.
Assign students to go to three public places (store, hospital, office building, clinic, etc.) and do the following: count the
number of handicapped spots in the parking lot, check to see how many bathrooms in each facility provide handicap
access, and provide a listing of other devices or architectural features that assist individuals who are handicapped, such
as Braille plates, automatic doors, etc. Ask students to write a one-page paper describing these features as well as any
deficiencies.
Ask students to break into two groups; one group of students will play the role of a hearing impaired patient, while the
other group will play the role of the medical assistant. Partner each hearing impaired patient with a medical assistant to
form a pair. Instruct the student portraying the deaf patient to insert ear plugs into both ears. Ask the student playing
the medical assistant to take a chief complaint from the student playing the part of the hearing impaired patient. Once
the activity is over, have each individual pair stand and share their observations of the activity and to elaborate on techniques that they used to communicate with each another.
Create a test or quiz using the test bank questions from Chapter 8 on the Instructor Resources CD.

27195_05_Lp01-09_395_452.indd 440

11/10/08 5:15:49 PM

Chapter 8 Assisting Patients With Special Needs

441

Go to the Web site www.virtualmedicalcentre.com and play videos that discuss topics such as depression, OCD,
schizophrenia, bipolar disease, and dementia so that students gain a better understanding of patients who suffer from
these disorders.

STUDENT PREPARATION AND ASSIGNMENTS


Read Chapter 8 in the text and complete the chapter assessment questions, field scenario questions, and StudyWARE
Challenge.
Watch the DVD Link and complete the questions at the end of the chapter.
Assign any StudyWARE activities and games not included in the StudyWARE Challenge.
Complete the Chapter 8 Workbook activities.
Complete the procedures in this chapter.

Procedure 8-1: Effectively Communicate with Patients from Different Cultures

Procedure 8-2: Effectively Communicate with Sight Impaired or Blind Patients

Procedure 8-3: Effectively Communicate with Hearing Impaired or Deaf Patients When an Interpreter Is Present

Procedure 8-4: Effectively Communicate with a Hearing Impaired or Deaf Patient Who Speech Reads

DEVELOPMENTAL OBJECTIVES
After completing this chapter, you should be able to:
1. Correctly spell and define the essential terms.
2. Explain the purpose of the Americans with Disabilities Act (ADA) and list what groups are included under this
provision.
3. List ways that the office can become compliant with ADA guidelines to assist sight impaired and blind patients.
4. Give four examples of auxiliary services and aids that can be used to assist the hearing impaired and deaf patients.
5. List six examples of accessible design features that can accommodate patients who have disabilities.
6. Describe obligations of the office in providing an interpreter for patients with limited English.
7. Explain the term cultural diversity and list some commonalities among similar cultures.
8. Describe why the implementation of many gestures is discouraged.
9. Explain the procedure for providing sighted guide assistance and briefly describe the procedures for accessing elevators,
stairways, and doorways.
10. List tips for working with older adults and pediatric patients.
11. List tips for working with mentally impaired and mentally challenged patients.

TEACHING AIDS AND EQUIPMENT


DVD player
Multimedia projector and the Chapter 8 instructor slides created in PowerPoint found on the Instructor Resources CD
Test bank questions for Chapter 8 found on the Instructor Resources CD
Computer with Internet to play some of the video clips mentioned in Activities for This Chapter
Blindfolds for students to experience what it is like to be blind and ear plugs for students to experience what it is like to
be deaf
Refer to the text for equipment/supply list for each procedure

27195_05_Lp01-09_395_452.indd 441

11/10/08 5:15:49 PM

442

Part 2 Chapter Lesson Plans

REFERENCES AND RESOURCES


Thomson Delmar Learnings Critical Thinking for Medical Assistants DVD Series. Program 4, Patient Education (Series ISBN
1-4018-3857-X)

WEB SITES
Computers Assisting the Handicapped: http://courses.cs.vt.edu/~cs3604/lib/Disabilities/murhpy.AT.html (Great Web
site that talks about how computers and the Internet are changing the lives of individuals with many types of disabilities)
American Foundation for the Blind: www.afb.org
Americans with Disabilities Act: www.ada.gov (Federal Web site for information on the ADA)
Guide Dogs for the Blind: www.guidedogs.com (Good information about guide dogs)
Braille Institute: www.brailleinstitute.org (Great Web site to teach more about providing sighted guide assistance when
working with blind patients)
Strategies for Teaching Students with Hearing Impairments: www.as.wvu.edu/~scidis/hearing.html#sect0 (Great Web
site for working with the deaf)
Virtual Medical Centre: www.virtualmedicalcentre.com (Great Web site with video clips that discusses conditions such
as dementia, bipolar disease, depression, etc.)

ANSWERS TO CRITICAL THINKING CHALLENGE TOOL BOXES


Note: Answers may vary from those suggested below.
Critical Thinking Challenge # 1:
A deaf patient schedules an appointment using relay services. You ask the patient if she will need the office to schedule an
interpreter for the visit. The patient states that she doesnt feel an interpreter will be necessary because her friend is going to
come with her and she just finished a sign language class at the community college.
1. List at least three reasons why using the friend in this scenario is not a good decision.
The friend is not seasoned enough to be able to interpret for the office visit.
The friend has not been trained in medical interpretation.
The patient may not be as likely to share information with the friend present in the room.
2. What would be an appropriate response for you to give to the patient?
Using someone who has just learned interpreting is probably not in your best interest. We can
provide a professional interpreter at no extra charge to you.
Critical Thinking Challenge # 2:
During the chief complaint segment of an in-office screening, an Iranian female patient complains of lower abdominal pain.
She also has some vaginal symptoms. You instruct the patient to remove her clothes from the waist down. The patient and her
husband appear quite distressed by the instructions and refuse to comply.
1. How should you respond to the couples distress?
Let the couple know that you respect their concerns. Reinforce why the patient needs to be examined. Let the couple know that you will be present during the exam and will assist in any way
that you can. If the couple still refuses, ask for assistance from the provider.
2. What types of reinforcement techniques can you use to help the patient and husband comply with the instructions?
See the response to question #1.

27195_05_Lp01-09_395_452.indd 442

11/10/08 5:15:50 PM

Chapter 8 Assisting Patients With Special Needs

443

Critical Thinking Challenge # 3:


An 83-year-old male patient with dementia appears confused by instructions that you give him for urine collection. His caregiver, his daughter, also tries to explain the procedure, but the patient just doesnt understand. The daughter volunteers to help
her father, but he quickly refuses the daughters assistance.
1. What are some things that you can do to help the patient with the instructions?
Give the patient a synopsis of what you need him to do and then provide each step separately
while assisting the patient or having the daughter assist the patient with each step, for instance:
Okay, Mr. Jones we are going to need to collect a urine sample from you today. Follow me to the
bathroom. . .Great, now I need you to wash and dry your hands. . .Great, now I need you to pull
down your pants. . . etc.
2. What might be an alternative if the patient refuses or just never comprehends the instructions?
Alert the provider of the patients refusal to obtain the specimen and ask the provider if he would
like the patient to be catheterized. This will vary according to office policy. The physician or nurse
may be the only personnel authorized to perform a catheterization. This may vary by state as well.

ANSWERS TO THE FIELD APPLICATION CHALLENGE


Note: Answers may vary from those suggested below.
Mrs. Adams, a blind patient, has an appointment to have some blood work performed today. She is accompanied by her guide
dog. You are ready to take the patient back to a blood drawing area.
1. How should you approach the patient in the reception area?
Approach the patient head on and gently call out the patients name.
2. Your office has a policy that no pets are allowed in the facility. Will you permit her to allow the dog to lead her back to the
blood drawing area? Why or why not?
Yes, by law guide dogs are permitted to escort the patient.
3. What should you do if you see people in the reception area trying to pet and talk to the dog?
Tactfully explain that the dog is working and that it is important to allow the dog to do his job.
4. Will the patient need sighted guide assistance?
No, because the dog will act as the patients guide.

ANSWERS TO THE CHAPTER ASSESSMENT


1. Examples of special needs patients include all but which of the following?
A. Blind patients
B. Deaf patients
C. Culturally diverse patients
D. Patients who work in health care
2. Which of the following acts/laws make it mandatory for offices to provide auxiliary aids or services to ensure effective
communication?
A. Title I of the American Disabilities Act
B. Title III of the American Disabilities Act
C. HIPAA
D. The Civil Rights Act

27195_05_Lp01-09_395_452.indd 443

11/10/08 5:15:50 PM

444

Part 2 Chapter Lesson Plans

3. When working with patients from other cultures, the medical assistant should:
A. use lots of gestures to enhance what is being stated.
B. ask the patient for the correct pronunciation of her name.
C. insist that the patient remove clothing regardless of the patients modesty concerns.
D. ask the patient about her religion to gain a better understanding of her beliefs.
4. Direct eye contact is considered disrespectful among:
A. Westerners.
B. Middle Easterners.
C. Latin Americans.
D. all of the above.
5. Which culture emphasizes interdependence over independence?
A. Western population
B. Middle Eastern population
C. Latin American population
D. Asian population
6. When pediatric patients are to receive an invasive procedure, the medical assistant should do all but which of the
following?
A. Tell them that it probably will not hurt.
B. Be honest and tell them that they may feel some discomfort.
C. Give them something to hold during the procedure like a stuffed animal or ball.
D. Explain the purpose of the procedure.
7. Patients with mental impairments should:
A. be treated with respect and kindness.
B. be addressed directly during the patient interview.
C. be assessed to determine their ability to communicate.
D. all of the above.
8. Which of the following is not a prerequisite to be considered a qualified interpreter under ADA guidelines?
A. Must be extremely fluent in the patients native language and English
B. Must be properly trained to provide interpretation in a medical facility
C. Must demonstrate competence in medical interpreting
D. Must be a U.S. citizen

ANSWERS TO THE DVD LINK


Note: Answers will vary but may include the following points.
1. Do you feel that it was appropriate for the medical assistant to start the visit without the interpreter? What device did the
medical assistant use to communicate until the interpreter arrived?
It is always best to wait until the interpreter is present. If this is not possible, you can use other
assistive devices to communicate with the patient until the interpreter arrives. The medical assistant used a computer to communicate with the patient until the interpreter arrived. (Always check
with the patient to make certain they are comfortable with the plan.)
2. How were the chairs set up during the educational session?
The chairs were set up in a triangular pattern. The patient was at the point and the medical assistant and the interpreter sat across from the patient.

27195_05_Lp01-09_395_452.indd 444

11/10/08 5:15:50 PM

Chapter 8 Assisting Patients With Special Needs

445

3. When the medical assistant gave instructions, who was he looking at: the patient or interpreter?
The patient
4. Do you feel that the medical assistant had good rapport with the patient and achieved his educational instruction goals?
Yes, the patient appeared to fully understand the medical assistants instructions and there
appeared to be good exchange between the patient and medical assistant.

27195_05_Lp01-09_395_452.indd 445

11/10/08 5:15:50 PM

27195_05_Lp01-09_395_452.indd 446

11/10/08 5:15:50 PM

Patient Education

TOPICS
1. Adult Education Principles
2. Settings and Procedures
3. Communication
4. Stimulating Patient Compliance
5. Topics for Education
6. Conducting Educational Sessions over the Telephone
7. Identifying Community Resources for the Patient

ABHES Content
VI.A.1.b.5.b

Caring for patients with special and specific needs

ABHES Competencies
VI.A.1.a.7.c
Teach patients methods of health promotion and disease
prevention
VI.A.1.a.3.e
Locate resources and information for patients and employers
CAAHEP Content
III.C.1.e.1
III.C.1.e.2
III.C.1.e.3
III.C.1.e.4

Principles of verbal and nonverbal communication


Recognition and response to verbal and nonverbal
communication
Adaptations for individualized needs
Applications of electronic technology

447

27195_05_Lp01-09_395_452.indd 447

11/10/08 5:15:50 PM

448

Part 2 Chapter Lesson Plans

CAAHEP Competencies
III.C.3.c.3.c
Provide instruction for health maintenance and disease prevention
III.C.3.c.3.d
Identify community resources

ONLINE COMPANION MATERIAL FOR THIS CHAPTER


2008 CAAHEP Content and Competencies Correlation Chart
Detailed chapter outline
Electronic competencies
Chapter updates
Access the Online Companion at www.delmarlearning.com/companions. Select Allied Health from the left menu, and then click on the link for
this book.

ACTIVITIES FOR THIS CHAPTER


Assign students to create an e-presentation that features a specific disease or condition. The e-presentation should
include links that provide the following information about the disease or condition: description, symptoms, diagnostic
testing, treatment, and follow-up information. Encourage students to use Web sites with wonderful graphics and animations, if possible. The e-presentation should include at least five different Web sites. The students should present their
e-presentations in front of the entire class. Emphasize the importance of structure and organization.
Organize an educational fair for members of your class. Invite students and staff members from all departments as well
as family members. Each student will select a health topic and create a booth that educates attendees about his or her
topic. Booths should look professional and include any or all of the following materials: tri-fold presentation board with
related information, models related to the topic, educational brochures, and computer or DVD player that features animations or information about the topic.
Assign students to contact a health educator from the community such as a diabetic educator or someone from the
STD division of the health department. Have each student shadow this individual for one or two presentations and
then write a paper summarizing his or her experience. The summary should include the topic, educational materials
used by the presenter, interaction observations, and likes and dislikes about the presentation.
Invite speakers from different organizations within the community who can assist patients with disease management,
support, and home assistance. Examples of agencies to contact include:

Public Health Department

Department of Health and Human Services

Battered Womens Shelters

Social service organizations

Mental health centers

STUDENT PREPARATION AND ASSIGNMENTS


Read Chapter 9 in the text and complete the chapter assessment questions, field scenario questions, and StudyWARE
Challenge.
Watch the DVD Link and complete the questions at the end of the chapter.
Assign any StudyWARE activities and games not included in the StudyWARE Challenge.
Complete the Chapter 9 Workbook activities.
Complete Procedure 9-1: Provide Instruction for Health Maintenance and Disease Prevention and Identify Community
Resources That Will Assist the Patient.

27195_05_Lp01-09_395_452.indd 448

11/10/08 5:15:50 PM

Chapter 9 Patient Education

449

DEVELOPMENTAL OBJECTIVES
After completing this chapter, you should be able to:
1. Correctly spell and define the essential terms.
2. List five characteristics of an adult learner.
3. List and define six barriers that impede learning during the educational encounter.
4. Describe the desired environment for educational sessions.
5. Describe the different types of senses used during educational sessions and give examples of tools that can be used for
each sense.
6. Describe three types of learners.
7. List and describe six different techniques that can be used to enhance learning and to help ascertain that the patient
comprehends the material.
8. List examples of various topics for patient education.
9. Describe common educational topics presented over the phone.
10. List different types of community resources that are available in assisting patients with overall health and well-being and
describe how to locate these resources.

TEACHING AIDS AND EQUIPMENT


DVD player
Multimedia projector and the Chapter 9 instructor slides created in PowerPoint found on the Instructor Resources CD
Test bank questions for Chapter 9 found on the Instructor Resources CD
Computer and Internet connection to show different educational Web sites
Refer to the text for equipment/supply list for each procedure

REFERENCES AND RESOURCES


Thomson Delmar Learnings Critical Thinking for Medical Assistants DVD Series. Program 4, Patient Education (Series ISBN
1-4018-3857-X)
Patient/Client Education, Christensen (ISBN 0-7668-2556-6)

WEB SITES
Familydoctor.org from the American Academy of Family Physicians: http://familydoctor.org/online/famdocen/
home.html (All major health topics)
Centers for Disease Control and Prevention: www.cdc.gov/vaccines/ed/patient-ed.htm (Vaccines)
Krames Patient Education: www.krames.com (Wide variety of topics; put in any topic you are looking for in regard to a
specific disease and condition, and you will get many references)
Major organizations, including the American Diabetes Association, American Heart Association, American Lung
Association, American Kidney Association, etc.

27195_05_Lp01-09_395_452.indd 449

11/10/08 5:15:50 PM

450

Part 2 Chapter Lesson Plans

ANSWERS TO CRITICAL THINKING CHALLENGE TOOL BOXES


Note: Answers may vary from those suggested below.
Critical Thinking Challenge # 1:
You are presenting an important educational session to the patient regarding cancer prevention and awareness. The patients
mother and sister both died from ovarian cancer. The patient brought her four-year-old niece to the session. The niece continually interrupts the patient throughout the session.
1. What are some activities that you can institute to keep the four-year-old busy during the educational session?
You may be able to give the niece some activity or coloring books to keep her busy during the
session. You may also ask her to draw some pictures.
Critical Thinking Challenge # 2:
You just finished a patient education session regarding hypertension with a 70-year-old patient who is a bit hard of hearing. He
is alone and doesnt appear to have a real interest in what you are presenting.
1. What may be contributing to the patients apparent lack of interest in the subject matter?
The patient may have a hearing problem or he may be in a state of denial or not understand the
seriousness of hypertension.
2. What techniques can you use to ensure that the patient comprehends the information?
Ask the patient to provide a brief summary of what he learned during the session or ask him a list
of questions that would help confirm his understanding of the main points.

ANSWERS TO THE FIELD APPLICATION CHALLENGE


Note: Answers may vary from those suggested below.
While speaking to the patient about a newly prescribed medication, you notice that the patient appears to be slightly agitated.
The patient tells you that he has a part-time job with limited benefits and that he doesnt qualify for any kind of prescription
coverage. There is no generic substitute for this drug, and it is important in the patients recovery.
1. What should be your first course of action?
Let the patient know that you will talk to the physician about his circumstances and see if there
are any alternative treatments available.
2. What are some additional actions that you may take to assist the patient in getting some financial relief?
If the physician feels that it is absolutely necessary for the patient to take this particular drug, you
may try contacting the drug company to see if they can provide stock samples until another resolution can be made. You can also contact social service agencies to see if they are able to assist
this gentleman.
3. What should you do if the patient refuses assistance and states that he doesnt want the drug and that everyone would be
much better off if he would die?
Do your best to reassure the patient that his thoughts are not valid. Definitely share his comments with the physician before the patient leaves to determine if the physician wants to meet
with him. He may need to be referred to a mental health agency.

ANSWERS TO THE CHAPTER ASSESSMENT


1. All of the following are basic theories regarding adult learning except:
A. Adults tend to integrate new information with past experiences and prior education.
B. Adults tend to prefer self-directed education rather than group involvement.

27195_05_Lp01-09_395_452.indd 450

11/10/08 5:15:50 PM

Chapter 9 Patient Education

451

C. Adults prefer the medical assistant to set the pace for their learning, controlling start and stop times.
D. A comfortable learning environment contributes to the successfulness of the session.
2. Barriers to education include:
A. time.
B. money.
C. interest.
D. all of the above.
3. Skills related to good verbal communication are:
A. echoing.
B. reassurance.
C. empathy.
D. summarization.
E. all of the above.
4. Brochures, DVDs, PowerPoints, slides, photos, handouts, models, and the Internet are excellent tools for which of the
following learning styles?
A. Visual
B. Audio
C. Tactile
D. All of the above
5. Tips for patient compliance include all of the following except:
A. Prevent family members from being in the room during the education session.
B. Look for indications that the patient understands and agrees with the material being presented.
C. Be a good role model.
D. Give the patient related supplies.

ANSWERS TO THE DVD LINK


Note: Answers may vary from those suggested below.
1. What did Jae Min do to make certain that she had a clear understanding of the physicians orders?
She reviewed them with the physician to clarify the information before calling the patient.
2. What did Jae Min need to check before giving out information to Mr. Wilsons wife?
She checked to see if Mrs. Wilson was listed on the privacy statement or on a release form stating
that she could receive PHI before discussing the patient with Mrs. Wilson.
3. Do you feel that Jae Mins accent contributed to Mr. Wilsons inability to understand her? If so, is there anything that Jae
Min can do to assist her with communicating over the phone?
It may have contributed to a degree; however, the patients hearing may have also contributed
to the patients inability to understand what the medical assistant was stating. Jae Min needs to
make certain that she slows down when she speaks and carefully enunciates her words. Jae Min
may want to consider making a list of words that people ask her to re-pronounce on a regular
basis and then work particularly hard on learning how to pronounce those words.

27195_05_Lp01-09_395_452.indd 451

11/10/08 5:15:51 PM

27195_05_Lp01-09_395_452.indd 452

11/10/08 5:15:51 PM

10

Principles of Infection
Control and OSHA Standards
TOPICS
1. The Infection Process
2. The Chain of Infection
3. Environmental Requirements for Microorganisms
4. Stages of Infection
5. The Bodys Mechanisms of Defense
6. Infection Control
7. Universal Blood and Body Fluid Precautions
8. Commonly Transmitted Bloodborne Diseases
9. OSHA Regulations
10. Exposure to Hazardous Chemicals
11. Safeguards in the Educational Environment

ABHES Content
VI.A.1.b.7.e.3

Professional office environment and safety

ABHES Competencies
VI.1.a.4.c
Apply principles of aseptic technique and infection control
VI.1.a.4.r
Practice standard precautions
VI.1.a.4.q
Dispose of biohazardous wastes
CAAHEP Content
III.c.1.g.1

Asepsis and infection control


453

27195_06_Lp10-18_453_508.indd 453

11/10/08 5:16:24 PM

454

Part 2 Chapter Lesson Plans

CAAHEP Competencies
III.C.3.b.1.a
III.C.3.b.1.e
III.C.3.b.1.d

Perform handwashing
Practice standard precautions
Dispose of biohazardous wastes

ONLINE COMPANION MATERIAL FOR THIS CHAPTER


2008 CAAHEP Content and Competencies Correlation Chart
Detailed chapter outline
Electronic competencies
Chapter updates
Access the Online Companion at www.delmarlearning.com/companions. Select Allied Health from the left menu, and then click on the link for
this book.

ACTIVITIES FOR THIS CHAPTER


Invite someone from the local health department or the company that takes care of your schools regulated waste to
conduct an in-service session on infection control or OSHA Standards for the class.
Assign students to research hand hygiene on the CDCs Web site (www.cdc.gov/mmwr/preview/mmwrhtml/rr5116a1.
htm), and then write a one-page paper on their findings.
To stress the importance of good hand hygiene, purchase a handwashing training kit that assists with training health
care workers to use the correct technique for handwashing. Most kits come with a special hand cleansing product as
well as an ultraviolet light so that students can see any areas that were missed during handwashing. Examples of kits
you may purchase include:

Glo Germ Products: www.glogerm.com

Globright: www.globright.com

Assign students to write a research paper or have them create a presentation using PowerPoint on one of the following
diseases or conditions:

Hepatitis (all types)

HIV and AIDS

MRSA

Tuberculosis

STUDENT PREPARATION AND ASSIGNMENTS


Read Chapter 10 in the text and complete the chapter assessment questions, field scenario questions, and StudyWARE
Challenge.
Watch the DVD Link and complete the questions at the end of the chapter.
Assign any StudyWARE activities and games not included in the StudyWARE Challenge.
Complete the Chapter 10 Workbook activities.
Complete the procedures in this chapter.

Procedure 10-1: Perform Medically Aseptic Handwashing

Procedure 10-2: Perform an Alcohol-Based Hand Rub

Procedure 10-3: Remove Contaminated Gloves

27195_06_Lp10-18_453_508.indd 454

11/10/08 5:16:27 PM

Chapter 10 Principles of Infection Control and OSHA Standards

455

DEVELOPMENTAL OBJECTIVES
After completing this chapter, you should be able to:
1. Correctly spell and define the essential terms.
2. List and define steps in the chain of infection.
3. List the five classifications of pathogenic organisms.
4. List and describe the six stages of infection.
5. Describe the inflammatory response.
6. Describe the four different types of immunity.
7. List and describe the different forms of vaccines available.
8. Explain the difference between medical and surgical asepsis.
9. List at least eight infectious diseases, their causative agents, and some signs and symptoms of each.
10. Explain the difference between Universal Precautions, Standard Precautions, and Transmission-Based Precautions.
11. Discuss the means of transmission for HIV as well as the stages of HIV infection and criteria used to diagnose AIDS.
12. List the five different types of hepatitis and how each is transmitted.
13. State the reason for the development of the Bloodborne Pathogen Standard and the areas governed by the standard.
14. List examples of body fluids and OPIM.
15. List the different types of PPE and state when each should be worn.
16. State the engineering and work practice controls necessary to minimize or eliminate employee exposure.
17. Describe the proper protocol for regulated waste disposal.
18. Be able to locate and use the MSDS manual.
19. List the safeguards to be followed in the classroom and educational laboratory.

TEACHING AIDS AND EQUIPMENT


DVD player
Multimedia projector and the Chapter 10 instructor slides created in PowerPoint found on the Instructor Resources CD
Test bank questions for Chapter 10 found on the Instructor Resources CD
PPE products that you can demonstrate in the classroom
OSHA manual/MSDS notebook
Fire extinguisher (to train students how to use a fire extinguisher)
Examples of OSHA labels and hazardous stickers
Refer to the text for equipment/supply list for each procedure

REFERENCES AND RESOURCES


Thomson Delmar Learnings Skills and Procedures for Medical Assistants DVD Series. Program 4, Infection Control (Series
ISBN 1-4018-3868-5)
Basic Infection Control for Healthcare Providers, Second Edition by Kennamer (ISBN 1-4180-1978-X)
The OSHA Handbook: The Guidelines for Compliance in Health Care Facilities by Acello (ISBN 0-7668-4059-X)

27195_06_Lp10-18_453_508.indd 455

11/10/08 5:16:27 PM

456

Part 2 Chapter Lesson Plans

WEB SITES
Centers for Disease Control and Prevention: www.cdc.gov/ncidod/dhqp (Infection control guidelines from the CDC)
OSHA: www.osha.gov (OSHA guidelines)
OSHA Books: http://osha.4ursafety.com (OSHA compliance products)
HIV InSite: http://hivinsite.ucsf.edu (Latest information on HIV)

ANSWERS TO CRITICAL THINKING CHALLENGE TOOL BOXES


Note: Answers may vary from those suggested below.
Critical Thinking Challenge # 1:
There is no sink with running water near your desk. Patients and other coworkers are continually stopping by your desk to drop
off and pick up items. You use the keyboard and pick up the phone several times per day.
1. What can you do to institute good hand hygiene without constantly leaving your desk?
Purchase antibacterial, waterless hand sanitizer to use at your desk.
Critical Thinking Challenge # 2:
While cleaning the exam room after a minor surgical procedure, you accidentally stick your thumb with a sharp probe that the
provider used on the patient. The patient is HIV positive.
1. What should be your first step in the process following a needlestick incident?
You first should immediately wash the area with an antibacterial soap and water and then report
the incident to your supervisor. (Note: All patients could be potentially infectious and all sticks
from contaminated needles should be treated in the same manner. Knowing that the patient is
already HIV positive will just expedite treatment.)
2. What prophylactic medications can be given to you?
Gamma globulin or other preventative medications
3. How long may it take before you produce a positive antibody test for HIV?
Two weeks to six months

ANSWERS TO THE FIELD APPLICATION CHALLENGE


Note: Answers may vary from those suggested below.
You are a medical assistant employed in a multiprovider family practice office. You are responsible for performing all venipunctures and processing all specimens for transport to the laboratory. While performing a routine venipuncture, you must
use several vacuum tubes before finding one that will fill completely with blood. After successfully obtaining the sample, you
prepare the specimen to be sent to the lab. As you are cleaning the work area, you inadvertently drop the full tube of blood into
the sharps container along with the defective tubes. The patient has already left the office.
1. What should you do first?
You should never take a chance on reaching into a sharps container even with an instrument. You
will need to call the patient and explain that the tube is not useable and that she will need to
come in and have her blood drawn again. It is important that you apologize to the patient for the
inconvenience.
2. How will you get a specimen to send to the lab?
The patient will have to come back in to have her blood redrawn.

27195_06_Lp10-18_453_508.indd 456

11/10/08 5:16:27 PM

Chapter 10 Principles of Infection Control and OSHA Standards

457

ANSWERS TO THE CHAPTER ASSESSMENT


1. The branch of science that studies the frequency and distribution of disease is:
A. demiology.
B. epidemiology.
C. pathology.
D. virology.
2. The stage of infection that produces no identifiable signs or symptoms is:
A. incubation stage.
B. acute stage.
C. invasion and multiplication stage.
D. prodromal stage.
3. The destruction of microorganisms before they enter the body is known as:
A. sterilization.
B. medical asepsis.
C. surgical asepsis.
D. disinfection.
4. Standard Precautions and Transmission-Based Precautions were developed by:
A. OSHA.
B. DDC.
C. CDC.
D. HHS.
5. Standard Precautions should be observed for:
A. tissue specimens.
B. CSF.
C. blood.
D. all of the above.
6. The disease posing the greatest risk to health care workers is:
A. AIDS.
B. HIV.
C. hepatitis C.
D. hepatitis B.
7. The primary means of spreading AIDS is:
A. sharing eating utensils.
B. sexual contact.
C. kissing.
D. none of the above.
8. Barriers used to protect health care workers from possible exposure to infectious agents are known as:
A. safety hoods.
B. Standard Precautions.
C. work practice controls.
D. PPE.
9. Regulated waste would include all of the following except:
A. contaminated sharps.
B. tubes containing blood.
C. glass slides.
D. contaminated dressings.

27195_06_Lp10-18_453_508.indd 457

11/10/08 5:16:27 PM

458

Part 2 Chapter Lesson Plans

ANSWERS TO THE DVD LINK


Note: Answers may vary from those suggested below.
1. What are two ways in which pathogens are transmitted from one individual to another? List examples of each.
Direct: Kissing, touching, sexual transmission, etc.
Indirect: Sneezing, coughing, touching door knobs, sharing eating and drinking utensils, etc.
2. How would you feel if someone like Mr. Breech came into your office and started coughing and contaminating the area in
which you are working?
Working as a medical assistant has its risks. I may not be thrilled that the patient is not taking
precautions to prevent exposure to myself or others but will do my best to tactfully educate the
patient on good precaution measures.
3. Obviously patients cant help it when they have to cough or sneeze, but what are two different precautions Mr. Breech
might have instituted to reduce the spread of microorganisms when he coughed?
Cover his mouth and nose with a tissue or other barrier; wear a mask to prevent spraying.

27195_06_Lp10-18_453_508.indd 458

11/10/08 5:16:27 PM

11

Basic Vital Signs and


Measurements
TOPICS
1. Introduction to the Patient
2. Height and Weight
3. Vital Signs

ABHES Content
VI.A.1.b.9.a

Basic clinical skills (e.g., vital signs)

ABHES Competencies
VI.A.1.a.4.d

Take vital signs

CAAHEP Competencies
III.C.3.b.4.b

Obtain vital signs

ONLINE COMPANION MATERIAL FOR THIS CHAPTER


2008 CAAHEP Content and Competencies Correlation Chart
Detailed chapter outline
Electronic competencies
Chapter updates
Access the Online Companion at www.delmarlearning.com/companions. Select Allied Health from the left menu, and then click on the link for
this book.

459

27195_06_Lp10-18_453_508.indd 459

11/10/08 5:16:27 PM

460

Part 2 Chapter Lesson Plans

ACTIVITIES FOR THIS CHAPTER


Plan a mini health fair at your institution that will include vital sign stations. This will allow students to take readings
from people other than their classmates.
Have students turn in weekly vital sign logs that require them to take at least 1015 sets of vitals per week. Participants
can be classmates, faculty and staff members from the institution, or friends and family members. Each set of vital
signs should be signed by the participant.
Invite externship supervisors or employers from the field to come and talk to the class about the importance of practicing vital signs and the expectations that sites place on vital signs.
Use the test bank on the Instructor Resources CD to create a pop quiz for this chapter.
Have students practice entering vital signs electronically by creating records for each of their classmates using
SynapseEHR.

STUDENT PREPARATION AND ASSIGNMENTS


Read Chapter 11 in the text and complete the chapter assessment questions, field scenario questions, and StudyWARE
Challenge.
Assign any StudyWARE activities and games not included in the StudyWARE Challenge.
Complete the Chapter 11 Workbook activities.
Complete the procedures in this chapter.

Procedure 11-1 Obtain the Height and Weight of an Adult Patient

Procedure 11-2: Obtain an Oral Body Temperature

Procedure 11-3: Obtain an Aural Body Temperature

Procedure 11-4: Obtain an Axillary Body Temperature

Procedure 11-5: Obtain a Temporal Artery Body Temperature

Procedure 11-6: Obtain a Radial Pulse Rate and Respiration Rate

Procedure 11-7: Obtain an Apical Pulse Rate

Procedure 11-8: Obtain a Blood Pressure Measurement Using the Palpatory Method

DEVELOPMENTAL OBJECTIVES
After completing this chapter, you should be able to:
1. Correctly spell and define the essential terms.
2. Explain the purpose for obtaining a height and weight.
3. List the vital signs, the reasons for obtaining them, and the normal ranges for each.
4. Describe the mechanisms that may cause variations in vital signs.
5. List the sites available for obtaining a temperature and the specific reasons for taking measurements at each site.
6. Identify the different pulse points.
7. Define blood pressure and the meaning of systole and diastole.

27195_06_Lp10-18_453_508.indd 460

11/10/08 5:16:27 PM

Chapter 11 Basic Vital Signs and Measurements

461

TEACHING AIDS AND EQUIPMENT


DVD player
Multimedia projector and the Chapter 11 instructor slides created in PowerPoint found on the Instructor Resources CD
Test bank questions for Chapter 11 found on the Instructor Resources CD
CD player to bring up the SynapseEHR 1.1 CD to enter vital signs in electronic format.
Refer to the text for equipment/supply list for each procedure

REFERENCES AND RESOURCES


Thomson Delmar Learnings Skills and Procedures for Medical Assistants DVD Series. Program 5, Taking Measurements and
Vital Signs (Series ISBN 1-4018-3868-5)
Vital Signs for Medical Assistants Interactive CD-ROM (ISBN 1-4018-7124-0)
Delmars Clinical Handbook, Second Edition, Krebs and Heller (ISBN 1-4018-3285-7)

ANSWERS TO CRITICAL THINKING CHALLENGE TOOL BOXES


Note: Answers may vary from the ones suggested below.
Critical Thinking Challenge # 1:
Refer to the Patient Perspective tool box on this page.
Patient Perspective: I have been watching my dietary intake and working out more for several months now. When I first started the exercise program, I was losing weight at a steady pace. Now my clothes are starting to fit better, but my weight is increasing. This is really getting frustrating.
Maybe I should just forget this fitness stuff.
1. What might be causing the patient to gain weight even though he may be spending so much time working out?
Fat may be turning to muscle. Because muscle weighs more than fat, the patient may temporarily
gain some weight before losing weight.
2. What can you do to encourage the patient not to give up?
Remind the patient that results are not always instantaneous and that he should start to notice
a difference in the near future. In the meanwhile, his clothes are fitting better and the increased
activity is helping his heart to function better, contributing to overall health and fitness.
Encourage the patient to communicate with you by e-mail and to give you weekly updates. Let
the patient know how proud you are of his new lifestyle choices.
3. Some patients may not come right out and tell their frustrations. What nonverbal signs might indicate that the patient is
getting frustrated?
Nonverbal signs may include sad expression, hands crossed, body slouched, a general distance
when talking to him, etc.
Critical Thinking Challenge # 2:
You work for a pediatrician. A mother calls and says her six-year-old daughter has been sick for the past two days, with a fever
ranging between 99F and 101F and that her temperature always seems to go up at night. The daughter isnt complaining of
any other symptoms, but the mother is concerned because of the night peaks.
1. What kind of information can you share with the mother that may help her feel more at ease?
Let the mother know that it is common for fevers to peak at night because of increased metabolism; however, also offer the mother an opportunity to bring her daughter in for an appointment
just to make certain that everything is alright (if it is office policy to do so).

27195_06_Lp10-18_453_508.indd 461

11/10/08 5:16:28 PM

462

Part 2 Chapter Lesson Plans

Critical Thinking Challenge # 3:


You are having great difficulty locating a patients radial pulse. You have attempted to count the pulse rate in both arms but
you just cant feel it.
1. What should you do?
Take an apical reading.
Critical Thinking Challenge # 4:
When performing a blood pressure on Mr. Timmons, you obtained a reading of 164/110. Dr. Wong retook the blood pressure
and obtained areading of 190/110. You are frustrated because you pumped the cuff up to 180 but didnt hear anything until
164.
1. What may be the reason for the discrepancy?
The patient may have what is known as an auscultatory gap or an occurrence in which Korotkoff
sounds diminish and become inaudible during phase two or phase three of the blood pressure
measurement.

ANSWERS TO THE FIELD APPLICATION CHALLENGE


Note: Answers may vary from those suggested below.
You are working in a family practice office and a regular patient, Mrs. Collins, age 72, walks in complaining of feeling dizzy with
a headache. You have assisted with her examinations many times in the past. She appears flushed and irritated. On the way to
the examination room, she stumbles slightly. Because she seems weak, you decide not to weigh her and to take her vitals in the
exam room.
Her blood pressure is elevated. She seems confused when you ask her about her medications. She was on an antihypertensive medication at the time of her last visit. She took a cab to the clinic for her appointment because she was afraid to drive.
No relatives are with her, although her daughter usually accompanies her.
1. What method of obtaining temperature would you suggest?
With her symptoms, it might be best to not put a thermometer in her mouth. You should take an
aural or temporal reading instead.
2. How much of an elevation in the blood pressure would be a cause for concern?
Anytime a patients BP reading is elevated there is cause for concern, especially when the patient
displays symptoms that could indicate a cardiac event or stroke. (Any reading that is 160/100 or
greater is very concerning!)
3. What information is of foremost importance to the provider?
The patients headache and unusual confusion, dizziness, and stumbling are of utmost importance.
We know that the patients blood pressure is elevated but we dont know the actual reading.
4. Would you leave her unaccompanied in the exam room?
No, you should not leave her unattended. Try to get the attention of another employee walking
by the room, or if the exam room has a phone, call someone to come and assist you. You need to
act in a quiet and calm manner so that you dont upset the patient or any other patients in the
practice.

27195_06_Lp10-18_453_508.indd 462

11/10/08 5:16:28 PM

Chapter 11 Basic Vital Signs and Measurements

463

ANSWERS TO THE CHAPTER ASSESSMENT


1. During the initial screening of the patient, the medical assistant observes:
A. appearance.
B. gait.
C. odor.
D. emotions.
E. all of the above.
F. B and C only.
2. Causes of heat production include all of the following except:
A. exercise.
B. shivering.
C. perspiration.
D. diet.
3. Which of the following pulse rates would be considered as normal for an adult?
A. 120 BPM
B. 40 BPM
C. 102 BPM
D. 68 BPM
4. The average respiratory rate for an adult is:
A. 1214 per minute.
B. 4244 per minute.
C. 1620 per minute.
D. 2428 per minute.
5. The respiratory rate can be affected by all except:
A. physical activity.
B. emotions.
C. medications.
D. diet.
6. The pulse pressure when taking a blood pressure indicates the:
A. difference between the palpable systolic and diastolic pressures.
B. difference between the palpable systolic and auditory systolic pressures.
C. difference between the palpable diastolic and auditory diastolic pressures.
D. difference between the auditory systolic and diastolic pressures.
7. When taking an apical pulse, the stethoscope chest piece is placed in what location?
A. Between the third and fourth ribs
B. Over the apex of the heart at the fifth intercostal space
C. At the midaxillary line of the fifth intercostal space
D. At the midnipple line between the third and fourth ribs
8. If the medical assistant notices that there is an irregularity when taking a pulse, the pulse should be taken for what additional length of time?
A. 15 seconds
B. 30 seconds
C. 1 minute
D. 2 minutes

27195_06_Lp10-18_453_508.indd 463

11/10/08 5:16:28 PM

You might also like